2015 pharmacotherapy specialty examination review course...

43
2015 Pharmacotherapy Specialty Examination Review Course: Complex Breast Cancer Case Helen M. Marshall, Pharm.D., BCPS, BCOP Clinical Pharmacist, Hematology/Oncology Seattle Cancer Care Alliance/University of Washington Medical Center Clinical Assistant Professor University of Washington School of Pharmacy Seattle, Washington Learning Objectives: At the end of the presentation, the pharmacist should be able to: Correctly answer case-based questions about appropriate treatment and monitoring of a complex patient with multiple conditions, including breast cancer, febrile neutropenia, and pulmonary embolism. Develop a plan to manage pain and nausea/vomiting in a patient with cancer. Determine how to manage drug-drug and drug-disease interactions in a cancer patient. Discuss ethical, cultural, health literacy, legal, economic, quality of life, or safety issues in this population. Identify and recommend appropriate resource organizations/groups to assist a specific patient. Format: This session will use a series of audience response questions to engage the audience and to prepare participants to answer similar questions on the pharmacotherapy board certification examination. The speaker will discuss drug therapy management in a cancer patient and discuss the rationale and guidelines driving therapeutic decisions. Premise: You are a clinical pharmacist in a large, 800-bed institution. You also participate in a continuity clinic one afternoon per week in an ambulatory, multidisciplinary breast cancer clinic. You are responsible for drug therapy monitoring in both settings and ensuring patient safety for both chemotherapy and supportive care. You have online access to an integrated medical record that is shared in both the inpatient and outpatient settings. NOTE: NCCN Clinical Practice Guidelines in Oncology (NCCN Guidelines®) are referenced with permission from the National Comprehensive Cancer Network® (NCCN®). To view the most recent and complete version of the guideline, go online to NCCN.org. NATIONAL COMPREHENSIVE CANCER NETWORK®, NCCN®, NCCN GUIDELINES®, and all other NCCN Content are trademarks owned by the National Comprehensive Cancer Network, Inc. ______________________________________________________________________________________________ ©2015 American Society of Health-System Pharmacists, Inc. All rights reserved. 1

Upload: others

Post on 10-Aug-2020

9 views

Category:

Documents


0 download

TRANSCRIPT

Page 1: 2015 Pharmacotherapy Specialty Examination Review Course ...elearning.ashp.org/Files/Org/c52850f8e2e14ca8b14b... · prepare participants to answer similar questions on the pharmacotherapy

2015 Pharmacotherapy Specialty Examination Review Course: Complex Breast Cancer Case

Helen M. Marshall, Pharm.D., BCPS, BCOP Clinical Pharmacist, Hematology/Oncology

Seattle Cancer Care Alliance/University of Washington Medical Center Clinical Assistant Professor

University of Washington School of Pharmacy Seattle, Washington

Learning Objectives: At the end of the presentation, the pharmacist should be able to:

• Correctly answer case-based questions about appropriate treatment and monitoring of a complex patient with multiple conditions, including breast cancer, febrile neutropenia, and pulmonary embolism.

• Develop a plan to manage pain and nausea/vomiting in a patient with cancer. • Determine how to manage drug-drug and drug-disease interactions in a cancer patient. • Discuss ethical, cultural, health literacy, legal, economic, quality of life, or safety issues in this

population. • Identify and recommend appropriate resource organizations/groups to assist a specific patient.

Format: This session will use a series of audience response questions to engage the audience and to prepare participants to answer similar questions on the pharmacotherapy board certification examination. The speaker will discuss drug therapy management in a cancer patient and discuss the rationale and guidelines driving therapeutic decisions. Premise: You are a clinical pharmacist in a large, 800-bed institution. You also participate in a continuity clinic one afternoon per week in an ambulatory, multidisciplinary breast cancer clinic. You are responsible for drug therapy monitoring in both settings and ensuring patient safety for both chemotherapy and supportive care. You have online access to an integrated medical record that is shared in both the inpatient and outpatient settings. NOTE: NCCN Clinical Practice Guidelines in Oncology (NCCN Guidelines®) are referenced with permission from the National Comprehensive Cancer Network® (NCCN®). To view the most recent and complete version of the guideline, go online to NCCN.org. NATIONAL COMPREHENSIVE CANCER NETWORK®, NCCN®, NCCN GUIDELINES®, and all other NCCN Content are trademarks owned by the National Comprehensive Cancer Network, Inc.

______________________________________________________________________________________________ ©2015 American Society of Health-System Pharmacists, Inc. All rights reserved.

1

Page 2: 2015 Pharmacotherapy Specialty Examination Review Course ...elearning.ashp.org/Files/Org/c52850f8e2e14ca8b14b... · prepare participants to answer similar questions on the pharmacotherapy

PATIENT CASE #1 Date: June 1, 2015 Initials GE DOB/Age

53 Sex F

Race/Ethnicity Caucasian

Source Patient and medical records

Chief Complaint/History of Present Illness: “I feel awful. I’m very short of breath and I can’t stop vomiting.” The patient is being treated for breast cancer and presents to the ED with acute onset of chest pain and shortness of breath. She also feels dizzy and weak. She has been unable to eat for the past day. Triage and work up begin immediately.

Past Medical History (major illnesses and surgeries)

From Medical Record Hypertension Depression Hypothyroidism Breast Cancer, Stage IIB

• Diagnosed January 2015 in her right breast • Stage IIB (T2 N1 M0) • ER and PR positive • HER2 negative • Postmenopausal (menopause at age 51)

Breast Cancer Treatment to Date

• Surgery ▫ Mastectomy with axillary lymph node dissection ▫ 4 positive lymph nodes

• Radiation ▫ Chest wall and regional lymph nodes

• Chemotherapy ▫ Dose-dense AC (Adriamycin [doxorubicin] and cyclophosphamide) ▫ Doxorubicin 60 mg/m2 IV day 1 ▫ Cyclophosphamide 600 mg/m2 IV day 1 ▫ Every 14 days X 4 cycles ▫ Dose-dense AC requires growth factor support ▫ Followed by paclitaxel ▫ Paclitaxel 80 mg/m2 IV day 1 weekly X 12 cycles

• GE received cycle 2 of doxorubicin and cyclophosphamide 2 days ago

______________________________________________________________________________________________ ©2015 American Society of Health-System Pharmacists, Inc. All rights reserved.

2

Page 3: 2015 Pharmacotherapy Specialty Examination Review Course ...elearning.ashp.org/Files/Org/c52850f8e2e14ca8b14b... · prepare participants to answer similar questions on the pharmacotherapy

Current Prescription/OTC Medications Start Date Drug Name/Strength/Regimen Indication 3/1999 Hydrochlorothiazide 25 mg orally once daily HTN 6/2002 Lisinopril 20 mg orally once daily HTN 1/1985 Multivitamin orally once daily Supplementation 3/2005 Calcium citrate 500 mg / cholecalciferol 400

units orally twice daily Supplementation/ Osteoporosis prevention

3/1997 Paroxetine 30 mg orally once daily Depression 11/1981 (last adjusted 3/2010)

Levothyroxine 88 mcg orally once daily Hypothyroidism

5/2015 Ondansetron 8 mg orally every 8 hr prn Nausea/vomiting 5/2015 Prochlorperazine 10 mg orally every 6 hr

prn Nausea/vomiting

5/2015 Lorazepam 0.5-1 mg orally every 6 hr prn Nausea/vomiting; anxiety 5/2015 Ondansetron 8 mg IV and dexamethasone

10 mg IV every 2 weeks pre-chemotherapy Prevention of nausea and vomiting

5/2015 Doxorubicin 60mg/m2 IV and cyclophosphamide 60mg/m2 IV every 2 weeks

Adjuvant breast cancer treatment

Vaccinations: Influenza vaccine fall 2014 Pharmacy(ies) Used: local community pharmacy, cancer center

RX Payment: Private (15% co-insurance) Meds Admin by: Self Drug Allergies/Adverse Effects: NKDA Family Medical History: Father: MI s/p CABG X 2 at age 55; Grandmother has a history of stage I breast cancer s/p mastectomy with reconstruction and no recurrence; Brother has MS Social History

Residence: lives at home w/ husband

Occupation: Elementary school teacher

Smoking: Previously smoked 1 ppd x 2 years in college

EtOH: 1-2 glasses of wine with dinner 2-3 times/week

Illicit Drugs: Marijuana in college only Diet: Vegetarian Education: college graduate Family/Social Environment: Lives with

husband; 1 son in his 20s (graduate school)

Review of Systems: Per HPI /PE

______________________________________________________________________________________________ ©2015 American Society of Health-System Pharmacists, Inc. All rights reserved.

3

Page 4: 2015 Pharmacotherapy Specialty Examination Review Course ...elearning.ashp.org/Files/Org/c52850f8e2e14ca8b14b... · prepare participants to answer similar questions on the pharmacotherapy

Objective Data (observations/vital signs/physical examination/labs)

General: Ill appearing female that is acutely SOB; A&0 x 3 RR 25/min, BP 90/52 mm Hg, HR 105 bpm, Temp 98.4 °F Wt 76 kg, Ht 166 cm Physical Exam – Diminished breath sounds on the left and right, left > right; tachycardic; skin and oral mucous membranes are dry. All other systems are negative/noncontributory. Laboratory Tests (drawn in ED) – Na 129 mEq/L K 3.3 mEq/L Cl 100 mEq/L CO2 32 mEq/L BUN 25 mg/dL SCr 1.2 mg/dL Glucose 95 mg/dL AST 27 units/L ALT 18 units/L Alk Phos 105 units/L T bili 0.8 mg/dL TP 7.1 g/dL Albumin 3.2 g/dL WBC 1.7 X 109/L Hgb 11.2 g/dL Hct 30% Plts 96 X 109/L ANC 0.6 X 109/L CK Index 1 CK-MB 1 ng/mL Total CK 41 ng/mL Troponin 0.2 ng/mL

______________________________________________________________________________________________ ©2015 American Society of Health-System Pharmacists, Inc. All rights reserved.

4

Page 5: 2015 Pharmacotherapy Specialty Examination Review Course ...elearning.ashp.org/Files/Org/c52850f8e2e14ca8b14b... · prepare participants to answer similar questions on the pharmacotherapy

Presentation Questions

1. Which of the following should be monitored for our patient on this chemotherapy regimen?

a. Cognitive function b. Pulmonary function c. Cardiac function d. Thyroid function

2. Which of the following is a risk factor for venous thromboembolism (VTE) in GE?

a. Febrile neutropenia b. Nausea and vomiting c. Presence of central venous catheter d. Hypertension

3. Which of the following therapies should be initiated in GE for her pulmonary embolism?

a. Warfarin 5 mg orally daily b. Apixaban 10 mg orally twice daily c. Dalteparin 12,500 units subcut every 24 hr d. Enoxaparin 80 mg subcut every 12 hr

4. What is the appropriate antiemetic regimen for GE while receiving treatment with dose-dense AC?

a. Ondansetron 8 mg IV and dexamethasone 12 mg IV day 1 followed by dexamethasone 4 mg PO BID days 2-4

b. Fosaprepitant 150 mg IV, ondansetron 8 mg IV, and dexamethasone 12 mg IV day 1 followed by dexamethasone 4 mg PO BID days 2-4

c. Fosaprepitant 150 mg IV, ondansetron 8 mg IV, and dexamethasone 12 mg IV day 1 d. Palonosetron 0.25 mg IV and dexamethasone 10 mg IV day 1 followed by dexamethasone 4 mg

PO BID days 2-4

5. Which of the following is a therapeutic option for anticoagulation in a patient with suspected HIT?

a. Argatroban b. Dalteparin c. Dabigatran d. Clopidogrel

6. Which of the following should be initiated as empiric therapy for febrile neutropenia in GE?

a. Imipenem 500 mg IV every 6 hours b. Ciprofloxacin 500 mg PO every 8 hours and clindamycin 450 mg PO every 8 hours c. Ciprofloxacin 500 mg PO every 8 hours and amoxicillin/clavulanate 500 mg PO every 8 hours d. Ciprofloxacin 500 mg IV every 8 hours

______________________________________________________________________________________________ ©2015 American Society of Health-System Pharmacists, Inc. All rights reserved.

5

Page 6: 2015 Pharmacotherapy Specialty Examination Review Course ...elearning.ashp.org/Files/Org/c52850f8e2e14ca8b14b... · prepare participants to answer similar questions on the pharmacotherapy

7. Which of the following considerations enters into the decision about whether vancomycin should be added to GE’s regimen?

a. Guidelines recommending empiric vancomycin for all patients with FN b. Presence of a central venous catheter c. Prior use of ciprofloxacin prophylaxis d. Reported symptom of cough

8. GE’s oncologist asks you for a conversion to a long-acting opioid because GE has escalated her oxycodone use to 5 mg every 3 hours around the clock and is still reporting poor pain control. Which of the following is the best regimen for GE?

a. Fentanyl TDS 100 mcg/hr every 72 hours b. Morphine SR 45 mg po every 8 hours c. Morphine SR 15 mg po every 12 hours d. Morphine SR 30 mg po every 12 hours

9. GE’s oncologist follows your advice and initiates the long acting pain regimen that was

recommended. Which of the following is the best breakthrough pain regimen for her?

a. Morphine IR 15 mg po every 1-2 hours b. Morphine IR 15 mg po every 4-6 hours c. Morphine IR 30 mg po every 6 hours d. Oxycodone 2.5 mg po every 6 hours

10. Which of GE’s medications poses a concern if initiating therapy with tamoxifen?

a. Lisinopril b. Levothyroxine c. Paroxetine d. Morphine

11. Which of the following is a possible treatment option for GE if her cancer returns (metastatic to the

bone) during treatment with tamoxifen?

a. Exemestane and everolimus b. Ado-trastuzumab emtansine c. Pertuzumab, trastuzumab, and paclitaxel d. Tamoxifen (complete 5 years)

______________________________________________________________________________________________ ©2015 American Society of Health-System Pharmacists, Inc. All rights reserved.

6

Page 7: 2015 Pharmacotherapy Specialty Examination Review Course ...elearning.ashp.org/Files/Org/c52850f8e2e14ca8b14b... · prepare participants to answer similar questions on the pharmacotherapy

Additional Information on Breast Cancer Febrile Neutropenia Pearls • A clinical practice guideline for the use of antimicrobial agents in patients with febrile neutropenia

and cancer was published by the Infectious Diseases Society of America (IDSA) in 2011. Some of the changes in this guideline include an emphasis on risk stratification of patients with FN, improved guidance on which patients benefit from prophylaxis (antibacterial, antiviral, and antifungal), and discussion of the use of empiric vs. preemptive antifungal therapy and emerging resistance problems.

• Main algorithms are below:

Frefield AG, Bow EJ, Sepkowitz KA et al. Clinical practice guideline for the use of antimicrobial agents in neutropenic patients with cancer: 2010 update by the Infectious Disease Society of America. Clin Infect Dis. 2011; 52:e56-93. http://www.uphs.upenn.edu/bugdrug/antibiotic_manual/idsaneutropenicfever2010.pdf (accessed 2015 May 19). Clinical infectious diseases by Infectious Diseases Society of America. Reproduced with permission of Oxford University Press in the format Journal via Copyright Clearance Center.

______________________________________________________________________________________________ ©2015 American Society of Health-System Pharmacists, Inc. All rights reserved.

7

Page 8: 2015 Pharmacotherapy Specialty Examination Review Course ...elearning.ashp.org/Files/Org/c52850f8e2e14ca8b14b... · prepare participants to answer similar questions on the pharmacotherapy

Frefield AG, Bow EJ, Sepkowitz KA et al. Clinical practice guideline for the use of antimicrobial agents in neutropenic patients with cancer: 2010 update by the Infectious Disease Society of America. Clin Infect Dis. 2011; 52:e56-93. http://www.uphs.upenn.edu/bugdrug/antibiotic_manual/idsaneutropenicfever2010.pdf (accessed 2015 May 19). Clinical infectious diseases by Infectious Diseases Society of America. Reproduced with permission of Oxford University Press in the format Journal via Copyright Clearance Center. • Modifications to initial antibiotics based on resistance.

o MRSA consider early addition of vancomycin, linezolid or daptomycin o VRE consider early addition of linezolid or daptomycin o ESBLs consider early use of a carbapenem o KPCs consider early use of polymyxin-colistin or tigecycline

• Duration of therapy is dictated by organism/site if documented infection; or at least until ANC recovers (>0.5 thou/microL) or longer depending on clinical situation.

______________________________________________________________________________________________ ©2015 American Society of Health-System Pharmacists, Inc. All rights reserved.

8

Page 9: 2015 Pharmacotherapy Specialty Examination Review Course ...elearning.ashp.org/Files/Org/c52850f8e2e14ca8b14b... · prepare participants to answer similar questions on the pharmacotherapy

• ASCO published guidelines for antimicrobial prophylaxis and outpatient management of FN o In patients with FN, those that meet MASCC criteria, those in Talcott group 4, and without

additional risk factors may be managed with empiric fluoroquinolone and amoxicillin/clavulanate (clindamycin for pateints with a penicillin allergy)

• NCCN® also lists moxifloxacin as an outpatient option for low risk FN patients • Fluoroquinolone prophylaxis may be used for patients at high risk with expected duration of

neutropenia ≤ 0.1 thou/microL for > 7 days. • Empiric antifungal coverage for persistent/recurrent fever after 4-7 days of antibiotics for

neutropenia expected to be > 7 days; preemptive therapy is acceptable for high-risk patients. • Antifungal prophylaxis should be used

o Against Candida for allogeneic stem cell transplant patients or undergoing intensive reinduction or salvage chemotherapy for acute leukemia (fluconazole, itraconazole, voriconazole, micafungin or caspofungin)

o Against Aspergillus for patients ≥ 13 years of age undergoing intensive therapy for acute leukemia or myelodysplastic syndrome with high risk of invasive aspergillosis (use posaconazole). May also use antifungal prophylaxis pre-engraftment in transplant patients with history of invasive aspergillosis, prolonged neutropenia ≥ 14 days, or prolonged period of neutropenia prior to transplant.

• Antiviral Prophylaxis o Acyclovir for herpes simplex virus (HSV)-seropositive patients undergoing allogeneic stem

cell transplant or leukemia induction therapy o Yearly influenza vaccination for all patients being treated for cancer o No other routine prophylaxis, but appropriate treatment if infection develops

• Myeloid growth factors are not recommended for established fever and neutropenia • Central line-associated bloodstream infections (CLABSI) in patients with FN

o Differential time to positivity > 120 minutes after blood cultures drawn from both the central line and peripherally suggests a CLABSI

o Infections caused by Staphylococcus aureus, Pseudomonas aeruginosa, fungi or mycobacteria Remove the catheter Systemic therapy for at least 14 days

o If caused by coagulase-negative Staphylococci, may maintain central line and treat with systemic therapy with or without antibiotic lock therapy

o Use duration of therapy of 4-6 weeks for deep tissue infection, endocarditis, septic thrombosis or persistent bacteremia/fungemia > 72 hours after catheter removal

• IDSA guidelines are also available (see reference list) for the treatment of candidiasis (2009) and catheter-related infections (2009).

Anticoagulation Pearls • Oncology patients should be provided education about the signs and symptoms of VTE • Anticoagulation should not be used to extend survival in patients with cancer in the absence of

other indications • At this time, there are insufficient data to recommend the use of dabigatran, rivaroxaban, and

apixaban in oncology patients. • Low molecular weight heparin (LMWH) remains the cornerstone of therapy.

______________________________________________________________________________________________ ©2015 American Society of Health-System Pharmacists, Inc. All rights reserved.

9

Page 10: 2015 Pharmacotherapy Specialty Examination Review Course ...elearning.ashp.org/Files/Org/c52850f8e2e14ca8b14b... · prepare participants to answer similar questions on the pharmacotherapy

• Anti-factor Xa monitoring is not routinely recommended, but it is used by some clinicians. The goal trough concentration is < 0.5 units/mL. You may consider this type of monitoring in special situations, such as changing renal function, pregnancy, or obesity.

• Routine monitoring of LMWH should include weight, SCr/CrCl, hematocrit and platelets. • In patients receiving extended anticoagulant therapy, the need for continued treatment should be

periodically reevaluated. • In patients with active cancer, extended anticoagulant therapy is recommended (beyond 3 months

of therapy) even when the risk for bleeding is high. • In patients with acute pulmonary embolism (PE) treated with LMWH, use a once-daily regimen

instead of a twice-daily regimen if possible to minimize the number of injections. • In patients with upper extremity DVT (UEDVT) associated with a central venous catheter, the

catheter should not be removed if it is functional and there is an ongoing need for it. • In cancer patients with UEDVT in whom the catheter is removed, 3 months of anticoagulant therapy

is suggested over extended therapy (Grade 2C). • In cancer patients with UEDVT in whom the catheter is not removed, anticoagulation should

continue for as long as the catheter remains (at least 3 months). • Hospitalized patients with cancer should receive prophylactic anticoagulation unless there is a

contraindication. Prophylactic anticoagulant therapy is recommended in the postoperative setting for 4 weeks, especially for high-risk abdominal or pelvic cancer surgery.

• Prophylactic anticoagulant therapy is recommended in high-risk ambulatory settings o Multiple myeloma patients receiving lenalidomide or thalidomide in combination with high-

dose dexamethasone or a multi-agent doxorubicin-containing regimen or multiple myeloma patients with other risk factors Prophylactic LMWH (i.e., enoxaparin 40 mg subcutaneously every 24 hours) Warfarin to target international normalized ratio (INR) 2-3

o Low-risk myeloma patients may receive prophylaxis with aspirin 81-325 mg once daily o Select the prophylactic agent based on cost, Food and Drug Administration (FDA)-approved

indication, ease of administration, reversibility, monitoring requirements, and presence of renal failure

• In the setting of heparin-induced thrombocytopenia (HIT), treatment with a direct thrombin inhibitor (DTI) or fondaparinux should overlap with warfarin therapy for a minimum of 5 days, and be continued until the INR is >2 for at least 24 hours.

• If argatroban is used, the target INR should be 4 before the drug is discontinued, and then the INR should be repeated within 3-6 hours after discontinuation of argatroban.

• Chromogenic factor X could be used to monitor the INR during DTI therapy because it is not affected by the DTI.

• Patients with cancer should be periodically assessed for venous thromboembolism (VTE) risk • Risk factors for VTE in malignancy

o General Patient Active cancer Advanced stage of cancer Types: brain, pancreas, stomach, bladder, gynecologic, lung, lymphoma,

myeloproliferative disease, kidney, metastatic Regional bulky lymphadenopathy with extrinsic vascular compression Familial and/or acquired hypercoagulability Medical comorbidities, including infection, renal disease, pulmonary disease,

congestive heart failure, or arterial thromboembolism Poor performance status

______________________________________________________________________________________________ ©2015 American Society of Health-System Pharmacists, Inc. All rights reserved.

10

Page 11: 2015 Pharmacotherapy Specialty Examination Review Course ...elearning.ashp.org/Files/Org/c52850f8e2e14ca8b14b... · prepare participants to answer similar questions on the pharmacotherapy

Advanced age o Treatment-related risk factors

Major surgery Central catheter/IV catheter Chemotherapy, especially bevacizumab or thalidomide/lenalidomide with

dexamethasone Exogenous estrogen, including hormone-replacement therapy, contraceptives,

tamoxifen/raloxifene, diethylstilbestrol o Modifiable risk factors

Smoking Obesity Exercise (lack of)

o Multiple myeloma patient risk factors M spike > 1.6 g/dL Hyperviscosity Progressive disease

o High-risk outpatients on chemotherapy (based on risk factor combinations) Active cancer with high risk of DVT (See above) Pre-chemotherapy platelet count > 300 thou/microL Pre-chemotherapy WBC > 11 thou/microL Hemoglobin <10 g/dL Use of erythropoiesis-stimulating agents BMI ≥ 35 kg/m2 Prior VTE

• NCCN® also has recommendations for managing therapeutic failure of anticoagulation, which does occur in the oncology population.

Chemotherapy Induced Nausea and Vomiting (CINV) Pearls

• Risk factors for CINV include o Young age o Sex (female increases risk) o Expectation of severe nausea o Chemotherapy

Dose Emetogenicity

o History of alcohol intake • There are three main sets of guidelines utilized in clinical practice (MASCC, ASCO, and NCCN®,

see references). There are some differences in each guideline in regards to classification of emetogenicity risk of each chemotherapeutic agent as well as recommendations for prophylaxis/treatment.

• Definitions: Emetic Risk Groups o High = Risk in > 90% of patients; cisplatin is a key example and frequently used in clinical

trials o Moderate = Risk in 30% to 90% of patients o Low = Risk in 10% to 30% of patients o Minimal – Risk in <10% of patients

• The combination of anthracycline and cyclophosphamide was reclassified to highly emetogenic in the 2011 update of the ASCO guideline.

______________________________________________________________________________________________ ©2015 American Society of Health-System Pharmacists, Inc. All rights reserved.

11

Page 12: 2015 Pharmacotherapy Specialty Examination Review Course ...elearning.ashp.org/Files/Org/c52850f8e2e14ca8b14b... · prepare participants to answer similar questions on the pharmacotherapy

• The guidelines also provide guidance for radiation-induced nausea/vomiting. • Acute emesis occurs within 24 hours of treatment • Delayed emesis occurs at least 24 hours after treatment • Antiemetic treatment for patients receiving combination chemotherapy should be determined

based on the agent with the highest emetic risk. • NCCN® Principles of Emesis Control

o Prevention is the goal o Risk of N/V for patients receiving highly or moderately emetogenic regimens lasts 3 days

for high regimens and 2 days for moderate regimens; use prophylaxis throughout o Oral and intravenous 5HT3 antagonists are equally effective at appropriate doses o Consider the toxicity of the selected agents (e.g., constipation with 5HT3 antagonists) o Choice of antiemetic should be based on regimen emetogenicity, patient factors and

prior antiemetic history o Consider other factors causing N/V in patients with malignancy o Consider H2 antagonist or PPI use to decrease dyspepsia which can exacerbate nausea o Lifestyle measures (e.g., small, frequent meals) may be useful

• Recommendations for Prophylaxis Emetic Risk Category Acute Delayed Notes High NK1 Antagonist AND

5HT3 Antagonist AND Dexamethasone

Dexamethasone (and aprepitant if used)

NCCN®: Consider adding lorazepam, H2 antagonist or PPI; also provides an olanzapine-based regimen

Moderate Palonosetron AND Dexamethasone

Dexamethasone NCCN® notes as above

Low 5HT3 Antagonist OR Dexamethasone OR Dopamine Receptor Antagonist

No routine prophylaxis NCCN®: Consider adding lorazepam, H2 antagonist or PPI ASCO: dexamethasone is recommended

Minimal No routine prophylaxis No routine prophylaxis

• Dexamethasone and 5HT3 antagonist are both recommended for high-dose chemotherapy (frequently used in bone marrow transplant)

• The ASCO guideline focuses on intravenous therapy; oral chemotherapy is discussed in NCCN® and MASCC

• Highly emetogenic agents are listed in the slide set

______________________________________________________________________________________________ ©2015 American Society of Health-System Pharmacists, Inc. All rights reserved.

12

Page 13: 2015 Pharmacotherapy Specialty Examination Review Course ...elearning.ashp.org/Files/Org/c52850f8e2e14ca8b14b... · prepare participants to answer similar questions on the pharmacotherapy

• Moderately Emetogenic Agents MASCC NCCN® ASCO Intravenous Agents

Oxaliplatin Cytarabine > 1000 mg/m2 Carboplatin Ifosfamide Cyclophosphamide < 1500 mg/m2 Azacitidine Alemtuzumab Doxorubicin Daunorubicin Epirubicin Idarubicin Irinotecan Bendamustine Clofarabine

Aldesleukin > 12-15 million IU/m2 Amifostine >300 mg/m2 Arsenic trioxide Azacitidine Bendamustine Busulfan Carboplatin Carmustine ≤ 250mg/m

2 Clofarabine Cyclophosphamide ≤ 1500 mg/m2 Cytarabine > 200 mg/m2 Dactinomycin Daunorubicin Doxorubicin < 60 mg/m2 Epirubicin ≤ 90mg/m

2 Idarubicin Ifosfamide < 2000 mg/m2/dose Interferon alfa > 10 million IU/m2 Irinotecan Melphalan Methotrexate ≥ 250 mg/m2 Oxaliplatin Temozolomide

Azacitidine Alemtuzumab Bendamustine Carboplatin Clofarabine Cyclophosphamide < 1500 mg/m2 Cytarabine > 1000 mg/m2 Daunorubicin Doxorubicin Epirubicin Idarubicin Ifosfamide Irinotecan Oxaliplatin

Oral Agents Cyclophosphamide Temozolomide Vinorelbine Imatinib

Moderate to High: Altretamine Busulfan (≥ 4 mg/day) Crizotinib Cyclophosphamide (≥100 mg/m2/day) Estramustine Etoposide Lomustine (single day) Mitotane Procarbazine Temozolomide (≥75 mg/m2/day) Vismodegib

______________________________________________________________________________________________ ©2015 American Society of Health-System Pharmacists, Inc. All rights reserved.

13

Page 14: 2015 Pharmacotherapy Specialty Examination Review Course ...elearning.ashp.org/Files/Org/c52850f8e2e14ca8b14b... · prepare participants to answer similar questions on the pharmacotherapy

• Low Emetogenicity Agents MASCC NCCN® ASCO Intravenous Agents Paclitaxel

Docetaxel Mitoxantrone Topotecan Etoposide Pemetrexed Methotrexate Doxorubicin HCl liposome injection Temsirolimus Ixabepilone Mitomycin Gemcitabine Cytarabine < 1000 mg/m2 5-fluorouracil Bortezomib Cetuximab Trastuzumab Catumaxomab Panitumumab Temsirolimus

Ado-trastuzumab emtansine Amifostine ≤ 300mg/m2 Aldesleukin ≤ 12 million IU/m2 Brentuximab Cabazitaxel Carfilzomib Cytarabine (low dose) 100-200 mg/m2 Docetaxel Doxorubicin (liposomal) Eribulin Etoposide 5-fluorouracil Floxuridine Gemcitabine Interferon alfa >5 <10 million international units/m2 Methotrexate >50 mg/m2 <250 mg/m2 Mitomycin Mitoxantrone Omacetaxine Paclitaxel Paclitaxel-albumin Pemtrexed Pentostatin Pralatrexate Romidepsin Thiotepa Topotecan Ziv-aflibercept

5-fluorouracil Bortezomib Cabazitaxel Catumaxomab Cytarabine ≤ 1000 mg/m2 Docetaxel Doxorubicin (liposomal) Etoposide Gemcitabine Ixabepilone Methotrexate Mitomycin Mitoxantrone Paclitaxel Panitumumab Pemetrexed Temsirolimus Topotecan Trastuzumab

Oral Agents Capecitabine Tegafur-uracil Etoposide Sunitinib Fludarabine Everolimus Lapatinib Lenalidomide Thalidomide

Low to Minimal: Axitinib Bexarotene Bosunitinib Busulfan (<4 mg/day) Cabozantinib Capecitabine Chlorambucil Cyclophosphamide (<100 mg/m2/day) Dasatinib Dabrafenib Erlotinib Everolimus Fludarabine Gefitinib Hydroxyurea

______________________________________________________________________________________________ ©2015 American Society of Health-System Pharmacists, Inc. All rights reserved.

14

Page 15: 2015 Pharmacotherapy Specialty Examination Review Course ...elearning.ashp.org/Files/Org/c52850f8e2e14ca8b14b... · prepare participants to answer similar questions on the pharmacotherapy

Imatinib Lapatinib Lenalidomide Melphalan Mercaptopurine Methotrexate Nilotinib Pazopanib Pomalidomide Potaninib Regorafenib Ruxolitinib Sorafenib Sunitinib Temozolomide (≤75 mg/m2/day) Thalidomide Thioguanine Topotecan Trametanib Tretinoin Vandetanib Vemurafenib Vorinostat

• Breakthrough nausea and vomiting

o Occurs despite prophylaxis (re-evaluate) o Treatment is largely empiric, without strong data or consensus recommendations o Drug classes frequently used include phenothiazines and benzodiazepines o Consider adding an additional class of medication to the current regimen o ASCO suggests olanzapine, high-dose intravenous metoclopramide, benzodiazepine

or dopamine antagonist • Anticipatory nausea and vomiting

o Use most active, appropriate regimen for prophylaxis o Consider behavioral therapy/desensitization o Consider benzodiazepine

Pain Management Pearls • Note that charts and tables of analgesic agents with equianalgesic dosing information are variable.

Doses recommended were obtained from studies of single doses for acute pain, were not bi-directional, and did not account for inter-patient variability in response.

• In addition to pharmacologic treatment of cancer pain, interventional (e.g., celiac plexus block, placement of an intrathecal catheter), behavioral, cognitive, rehabilitative and integrative modalities for management of cancer pain are available.

• Prescription drug abuse has continued to rise in the United States, and many states are enacting legislation to promote the safe use of opioid analgesics. In many states, cancer patients are exempt from legal requirements, but legislation in some states may apply to cancer patients with non-cancer chronic pain (e.g., low back pain).

______________________________________________________________________________________________ ©2015 American Society of Health-System Pharmacists, Inc. All rights reserved.

15

Page 16: 2015 Pharmacotherapy Specialty Examination Review Course ...elearning.ashp.org/Files/Org/c52850f8e2e14ca8b14b... · prepare participants to answer similar questions on the pharmacotherapy

• There is no upper dose limit for opioids in cancer pain. The correct dose controls the patient’s pain and does not cause unacceptable side effects.

• Opioids to avoid in patients with cancer pain: meperidine, pentazocine, butorphanol, nalbuphine, and buprenorphine.

• Hydrocodone has been reclassified to CII (2014) • Requirements for risk evaluation and mitigation strategies (REMS) have been established by FDA for

some opioids, including the newer extended-release hydromorphone product (available at: http://www.exalgorems.com/) and all immediate-release transmucosal fentanyl products (available at: https://www.tirfremsaccess.com/TirfUI/rems/home.action). Prescribers, patients, and pharmacies must be registered for outpatient use of these products.

• Conversion to fentanyl transdermal systems (TDS) can be accomplished using morphine equivalents (do not use the following table to convert from fentanyl to another opioid):

24-hr Oral Morphine Dose (mg) Fentanyl TDS dose (mcg/hr)

60-134 25 135-224 50 225-314 75 315-404 100 405-494 125 495-584 150 585-674 175 675-764 200 765-854 225 855-944 250

945-1034 275 1035-1124 300

• Fentanyl TDS Pearls

o Patients need to be on an oral morphine equivalent of at least 60 mg daily to start using transdermal systems

o The onset of action is approximately 12 hours o Removal of the TDS results in continued fentanyl exposure for approximately 17 hours o The system should be removed and replaced by a fresh one every 72 hours o The TDS is useful in patients with chronic, stable pain as well as patients who cannot

swallow solid oral dosage forms (e.g., patients with head or neck cancer) o Counseling patients on proper use and disposal of the TDS is imperative

• Methadone Clinical Pearls o Useful and inexpensive agent for oncology patients o Thought to provide additional pain relief by antagonizing N-methyl d-aspartate (NMDA)

receptors in the dorsal horn of the spinal cord o Analgesic duration (approximately 4-8 hours) and drug half-life (15-30 hours) are very

different o Drug accumulation may occur o Equianalgesic conversion is not linear, and very challenging o Doses should not be titrated faster than once every 5 days o Prolongs the QTc interval on the ECG (see the article by Krantz MJ et al in the pain

management section of the reference list in this handout for suggested monitoring parameters)

______________________________________________________________________________________________ ©2015 American Society of Health-System Pharmacists, Inc. All rights reserved.

16

Page 17: 2015 Pharmacotherapy Specialty Examination Review Course ...elearning.ashp.org/Files/Org/c52850f8e2e14ca8b14b... · prepare participants to answer similar questions on the pharmacotherapy

• Opioid Rotation (this is a more complex and conservative example than that provided during the presentation, but it illustrates the point that conversion from one agent to another is not straightforward; see the article by Portenoy (Lancet) in the pain management section of the reference list in this handout)

o Step 1 Select agent based on previous patient experience, availability, cost, etc Calculate equianalgesic dose For opioids other than methadone and fentanyl, identify a 25-50% dose reduction

window lower than the calculated dose • For methadone, the dose reduction window is 75-90%, rarely converting to

methadone at doses higher than 100 mg per day • For transdermal fentanyl, use the package insert

Select dose closer to the lower bound (25%) or upper bound (50%) based on how applicable the conversion is to the regimen and patient

• Upper bound if patient is on a fairly high dose, is not white, elderly or medically frail

• Lower bound if otherwise and especially if switching route of administration o Step 2

On the basis of assessment of pain severity and other medical/psychosocial patient characteristics, increase or decrease the calculated dose by 15-30% to improve the likelihood that the initial dose will be effective and/or prevent adverse effects/withdrawal

Assess response and titrate as necessary to optimize outcome • ASCO released guidelines for prevention and management of chemotherapy-induced peripheral

neuropathy in survivors of adult cancer (2014). There is a lack of evidence for preventative therapy. The guidelines make a moderate recommendation for the use of duloxetine based on a randomized controlled trial for 5 weeks duration that resulted in a decrease in average pain intensity score in the duloxetine arm of 1.06 (95% CI, 0.72-1.40) vs. 0.34 (95% CI, 0.01-0.66) in the placebo arm (P = .003; effect size, 0.513).

Breast Cancer Pearls • Most common malignancy in women (approximately 231, 840 new cases expected in 2015), and

second leading cancer killer behind lung cancer. Mortality from breast cancer appears to be declining.

• Risk factors o Gender (please remember that breast cancer can occur in men) o Advanced age o Genetics (BRAC1/2, others) o Family history, personal history o Race (Caucasian women have slightly higher incidence, but African-American women have a

higher mortality; Asian, Hispanic, and Native-American women have a lower risk) o Dense breast tissue o Benign breast conditions o Lobular carcinoma in situ (LCIS) o Menarche at age < 12 and menopause at age > 55 (more hormonal exposure) o Diethylstilbestrol exposure o Increased hormone exposure

Nulliparity

______________________________________________________________________________________________ ©2015 American Society of Health-System Pharmacists, Inc. All rights reserved.

17

Page 18: 2015 Pharmacotherapy Specialty Examination Review Course ...elearning.ashp.org/Files/Org/c52850f8e2e14ca8b14b... · prepare participants to answer similar questions on the pharmacotherapy

Lack of breast feeding Oral contraceptives Hormone-replacement therapy

o Alcohol use o Obesity o Lack of physical activity

• Cancers may arise from the ducts or lobules (85-90% of invasive are ductal) • Test all tumors for estrogen receptor (ER), progesterone receptor (PR) and HER2 status • Treatment includes local therapy with surgery and/or radiation therapy and the treatment of

systemic disease with chemotherapy, endocrine therapy, biologic therapy, or combinations of these modalities.

• Factors that influence treatment include: o Tumor histology o Clinical and pathologic characteristics of the tumor o Axillary node status o Tumor hormone receptor content o Tumor HER2 status o Multi-gene testing o Patient factors: comorbid conditions, age, and menopausal status

• Patients stratified for treatment: o Pure noninvasive carcinomas: LCIS and ductal carcinoma in situ (DCIS) (Clinical Stage 0) o Operable, local-regional invasive carcinomas with or without associated noninvasive

carcinoma (Clinical Stage I, Stage IIA, Stage IIB, and some Stage IIIA) o Inoperable local-regional invasive carcinoma with or without associated noninvasive

carcinoma (Clinical Stage IIIB, Stage IIIC, and some Stage IIIA) o Metastatic (Clinical Stage IV) or recurrent carcinoma

• Survival Data (Based on 2001-2002 diagnoses, American Cancer Society)

Stage 5-Year Overall Survival (%)

0 93

I 88

IIA 81

IIB 74

IIIA 67

IIIB 41

IIIC 49

IV 15

• Treatment o LCIS

Surveillance Risk reduction counseling

o DCIS Lumpectomy + radiation (Category 1, but no survival advantage) Mastectomy +/- reconstruction Lumpectomy alone with observation

______________________________________________________________________________________________ ©2015 American Society of Health-System Pharmacists, Inc. All rights reserved.

18

Page 19: 2015 Pharmacotherapy Specialty Examination Review Course ...elearning.ashp.org/Files/Org/c52850f8e2e14ca8b14b... · prepare participants to answer similar questions on the pharmacotherapy

Consider tamoxifen for ER positive o Stage I, IIA, IIB, some IIIA

Mastectomy with axillary lymph node dissection OR breast-conserving therapy with lumpectomy, axillary dissection, and whole breast irradiation (Stages I, II)

Consider neoadjuvant therapy for large IIA, IIB and T3N1M0 tumors when patients want to undergo breast-conserving therapy (sentinel lymph node dissection should be completed before neoadjuvant therapy)

• Neoadjuvant pertuzumab and trastuzumab-based regimens should be added to chemotherapy for patients with HER2 positive tumors

Individualized endocrine therapy (if ER/PR positive), chemotherapy Node status

• 4 positive – high risk for recurrence prophylactic chest wall/regional lymph node radiation after mastectomy and chemotherapy (can be given with endocrine therapy, HER2-targeted therapy)

• 1-3 positive – controversial; radiation recommended for large (>5 cm) tumors and/or positive margins

• Node negative – radiation for large tumors and/or positive margins Adjuvant Therapy

• Data for chemotherapy not definitive over age 70 • Online decision making tool (www.adjuvantonline.com) can be used to

assess need for adjuvant therapy (endocrine, chemotherapy) • Gene expression profile (DNA microarray technology) may be used to guide

therapy; trials ongoing • Not needed for small tumors <0.5 cm with no nodal involvement • Lymph node negative, hormone receptor negative tumor > 1 cm

chemotherapy • Lymph node negative, hormone receptor positive tumor > 1 cm

chemotherapy and endocrine therapy • Lymph node positive chemotherapy (and endocrine therapy if hormone

receptor positive) • Endocrine receptor positive

o Adjuvant therapy should follow chemotherapy • Postmenopausal tamoxifen for 4.5-6 years or aromatase

inhibitor(AI) for 5 years. Consider tamoxifen for an additional 5 years.

• Women with a contraindication to or who are intolerant to or decline therapy with aromatase inhibitors: tamoxifen for 5 years or consider tamoxifen for up to 10 years

o Premenopausal tamoxifen X 5 years, consider an additional 5 years of tamoxifen Patients who become postmenopausal can complete 5

years of AI therapy, then consider an additional 5 years of tamoxifen

Data is emerging regarding the role of ovarian ablation • Chemotherapy – preferred regimens all evaluated in Phase III trials (NCCN®

designates preferred vs. other based on efficacy and toxicity) o Dose-dense AC followed by sequential paclitaxel every 2 weeks o Dose-dense AC followed by weekly paclitaxel

______________________________________________________________________________________________ ©2015 American Society of Health-System Pharmacists, Inc. All rights reserved.

19

Page 20: 2015 Pharmacotherapy Specialty Examination Review Course ...elearning.ashp.org/Files/Org/c52850f8e2e14ca8b14b... · prepare participants to answer similar questions on the pharmacotherapy

o TC (docetaxel and cyclophosphamide) o Many “other” options

• Adjuvant trastuzumab therapy for HER2-positive disease X 1 year for tumors > 1 cm

o AC followed by paclitaxel with trastuzumab for 1 year starting with the first dose of paclitaxel

o Consider docetaxel and carboplatin (TCH) for patiients with risk factors for cardiac toxicity

o Inoperable Stage III Invasive Tumors Neoadjuvant anthracycline-based chemotherapy with or without a taxane Preoperative trastuzumab if HER2-positive Total mastectomy with axillary lymph node dissection with or without delayed

breast reconstruction OR lumpectomy and axillary dissection No matter what surgery, patients are at high risk for recurrence and therefore

should receive adjuvant radiation Completion of chemotherapy course if not all given preoperatively Endocrine therapy and/or trastuzumab as above for those with hormone receptor-

positive disease and/or HER2-positive disease, respectively o Metastatic (Stage IV) or Recurrent

Local recurrence • Previous mastectomy only salvage surgery if possible and radiation • Previous mastectomy + radiation limited duration systemic therapy • Previous breast-conserving therapy mastectomy, axillary lymph node

dissection if not done prior • Systemic therapy using the principles of adjuvant therapy

Systemic disease • Endocrine therapy preferred due to improved toxicity profile in the

palliative setting • Patients are stratified by presence of bone metastases then hormone

receptor and HER2 status • Initiate bisphosphonate or RANK ligand therapy for those with bone disease

o Monitoring includes serum calcium, phosphate, magnesium, renal function

o Use calcium and vitamin D supplementation o Bone-modifying therapies associated with osteonecrosis of the jaw

(ONJ) • Postmenopausal endocrine therapy options: anastrozole, letrozole,

exemestane, tamoxifen, toremifene, fulvestrant, megestrol acetate, fluoxymesterone, ethinyl estradiol

o The combination of everolimus and exemestane can be utilized in patients that have failed nonsteroidal aromatase inhibitors

o The combination of palbociclib, a novel oral cyclin-dependent kinase (CDK) inhibitor that blocks transition from the G1 to S phase of the cell cycle, can be used in combination with letrozole in 1st line therapy (not yet in NCCN® Guidelines)

• Premenopausal options: tamoxifen, toremifene, leuprolide, goserelin, megestrol acetate, fluoxymesterone, ethinyl estradiol, oophorectomy

o For most following initial therapy with tamoxifen, ovarian suppression/ablation with endocrine therapy is appropriate

______________________________________________________________________________________________ ©2015 American Society of Health-System Pharmacists, Inc. All rights reserved.

20

Page 21: 2015 Pharmacotherapy Specialty Examination Review Course ...elearning.ashp.org/Files/Org/c52850f8e2e14ca8b14b... · prepare participants to answer similar questions on the pharmacotherapy

• Chemotherapy o For hormone receptor negative tumors not localized to bone,

symptomatic visceral metastases, refractory to endocrine therapy o NCCN® preferred single agents

Anthracyclines – doxorubicin, liposomal doxorubicin Taxanes – paclitaxel, docetaxel, albumin-bound paclitaxel Antimetabolites – capecitabine, gemcitabine Non-taxane microtubule inhibitors – vinorelbine, eribulin

o Preferred combination regimens CAF (cyclophosphamide, doxorubicin, fluorouracil) FEC (fluorouracil, epirubicin, cyclophosphamide) AC (doxorubicin, cyclophosphamide) EC (epirubicin, cyclophosphamide) CMF (cyclophosphamide, methotrexate, fluorouracil) Docetaxel and capecitabine GT (gemcitabine and paclitaxel) Gemcitabine and carboplatin Paclitaxel and bevacizumab (no OS benefit)

o Failure to achieve a tumor response with 3 sequential chemotherapy regimens or ECOG performance status ≥ 3 is an indication for supportive care

• Add HER2-targeted therapies for HER2-positive disease o Trastuzumab + pertuzumab + docetaxel in patients who are

treatment naïve (preferred, Category 1) (with paclitaxel, Category 2A)

o “Other 1st line options;” single agent trastuzumab or with chemotherapy

o Ado-trastuzumab emtansine in patients that have previously received trastuzumab (preferred)

o “Other” regimes for trastuzumab exposed patiens include pertzumab and transtuzumab (if no previous pertuzumab) +/- chemotherapy; capecitabine and lapatinib; lapatinib and letrozole

o Do not use trastuzumab in combination with AC because of high risk for cardiotoxicity (27% is considered to be too high for the palliative setting)

______________________________________________________________________________________________ ©2015 American Society of Health-System Pharmacists, Inc. All rights reserved.

21

Page 22: 2015 Pharmacotherapy Specialty Examination Review Course ...elearning.ashp.org/Files/Org/c52850f8e2e14ca8b14b... · prepare participants to answer similar questions on the pharmacotherapy

References and Recommended Readings Febrile Neutropenia 1. Frefield AG, Bow EJ, Sepkowitz KA et al. Clinical practice guideline for the use of antimicrobial agents

in neutropenic patients with cancer: 2010 update by the Infectious Disease Society of America. Clin Infect Dis. 2011; 52:e56-93. http://www.uphs.upenn.edu/bugdrug/antibiotic_manual/idsaneutropenicfever2010.pdf (accessed 2015 May 19).

2. National Comprehensive Cancer Network. NCCN® clinical practice guidelines in oncology: prevention and treatment of cancer-related infections. Version 2.2014. Available at: http://www.nccn.org/professionals/physician_gls/pdf/infections.pdf. Referenced with permission from the NCCN Clinical Practice Guidelines in Oncology (NCCN Guidelines®) for Prevention and Treatment of Cancer-Related Infections V.1.2014 © National Comprehensive Cancer Network, Inc 2014. All rights reserved. Accessed [February 16, 2015].

3. Mermel LA, Allon M, Bouza E et al. Clinical practice guidelines for the diagnosis and management of

intravascular catheter-related infection: 2009 update by the Infectious Disease Society of America. Clin Infect Dis. 2009; 49:1-45. http://www.ncbi.nlm.nih.gov/pubmed/19489710 (accessed 2012 May 31).

4. Pappas PG, Kauffman CA, Andes D et al. Clinical practice guidelines for the management of candidiasis: 2009 update by the Infectious Diseases Society of America. Clin Infect Dis. 2009; 48:503-35. http://www.ncbi.nlm.nih.gov/pubmed/19191635 (accessed 2014 May 21).

5. Klastersky J, Paesmans M, Rubenstein EB et al. The Multinational Association for Supportive Care in Cancer risk index: a multinational scoring system for identifying low-risk febrile neutropenic cancer patients. J Clin Oncol. 2000; 18:3038-51. http://www.ncbi.nlm.nih.gov/pubmed/10944139 (accessed 2014 May 21).

6. Walsh TJ, Anaissie EJ, Denning DW et al. Treatment of aspergillosis: clinical practice guidelines of the Infectious Diseases Society of America. Clin Infect Dis. 2008; 46:327-60. http://www.uphs.upenn.edu/bugdrug/antibiotic_manual/idsa-aspergilloisrx-2008guidelines.pdf (accessed 2014 May 21).

7. Flowers CR, Seidenfeld J, Bow EJ et al. Antimicrobial prophylaxis and outpatient management of

fever and neutropenia in adults treated for malignancy: American Society of Clinical Oncology clinical practice guideline. J Clin Oncol. 2013; 31:794-810. http://jco.ascopubs.org/content/31/6/794.long (accessed 2014 May 21)

Anticoagulation 1. Lee AYY, Levine MN, Baker RI et al. Low-molecular-weight heparin versus a coumarin for the

prevention of recurrent venous thromboembolism in patients with cancer. N Engl J Med. 2003; 349:146-53. http://www.nejm.org/doi/full/10.1056/NEJMoa025313 (accessed 2014 May 21).

______________________________________________________________________________________________ ©2015 American Society of Health-System Pharmacists, Inc. All rights reserved.

22

Page 23: 2015 Pharmacotherapy Specialty Examination Review Course ...elearning.ashp.org/Files/Org/c52850f8e2e14ca8b14b... · prepare participants to answer similar questions on the pharmacotherapy

2. Kearon C, Akl EA, Comerota AJ et al. Antithrombotic therapy for VTE disease: antithrombotic therapy and prevention of thrombosis, 9th Ed: American College of Chest Physicians evidence-based clinical practice guidelines. Chest. 2012; 141(2 suppl):419S-94S. http://www.ncbi.nlm.nih.gov/pubmed/22315268 (accessed 2014 May 21).

3. National Comprehensive Cancer Network. NCCN® clinical practice guidelines in oncology: venous thromboembolic disease. Version 2.2014. Available at: http://www.nccn.org/professionals/physician_gls/pdf/vte.pdf. Referenced with permission from the NCCN Clinical Practice Guidelines in Oncology (NCCN Guidelines®) for Venous Thromboembolic Disease V.2.2014 © National Comprehensive Cancer Network, Inc 2014. All rights reserved. Accessed [February 16, 2015].

4. Lyman GH, Khorana AA., Kuderer NM et al. Venous thromboembolism prophylaxis and treatment in

patients with cancer: American Society of Clinical Oncology clinical practice guideline update. J Clin Oncol. 2013; 31:2189-204. http://jco.ascopubs.org/content/31/17/2189.long (accessed 2014 May 21).

5. Linkins LA, Dans AL, Moores LK et al. Treatment and prevention of heparin-induced

thrombocytopenia: antithrombotic therapy and prevention of thrombosis, 9th Ed: American College of Chest Physicians evidence-based clinical practice guidelines. Chest. 2012; 141(2 suppl):495S-530S. http://www.ncbi.nlm.nih.gov/pubmed/22315270 (accessed 2014 May 21).

6. Lo GK, Juhl D, Warkentin TE et al. Evaluation of pretest clinical score (4T’s) for the diagnosis of heparin-induced thrombocytopenia in two clinical settings. J Thromb Haemost. 2006; 4:759-65. http://www.ncbi.nlm.nih.gov/pubmed/16634744 (accessed 2014 May 21).

Nausea and Vomiting 1. Basch E, Pestrud AA, Hesketh PJ et al. Antiemetics: American Society of Clinical Oncology clinical

practice guideline update. J Clin Oncol. 2011; 29:4189-98. http://jco.ascopubs.org/content/29/31/4189.long (accessed 2014 May 21)

2. Gralla RJ, Rolia F, Tonato M et al. MASCC/ESMO antiemetic guidelines 2013. Available

at: http://www.mascc.org/antiemetic-guidelines. 3. National Comprehensive Cancer Network. NCCN® clinical practice guidelines in oncology:

antiemesis. Version 2.2014. Available at: http://www.nccn.org/professionals/physician_gls/pdf/antiemesis.pdf. Referenced with permission from the NCCN Clinical Practice Guidelines in Oncology (NCCN Guidelines®) for Antiemesis V.2.2014 © National Comprehensive Cancer Network, Inc 2014. All rights reserved. Accessed [February 16, 2015].

4. Hesketh PJ. Chemotherapy induced nausea and vomiting. N Engl J Med. 2008; 358:2482-

94. http://www.motilitysociety.org/clinician/manuscripts/drugNV_nejm.pdf (accessed 2014 May 21)

______________________________________________________________________________________________ ©2015 American Society of Health-System Pharmacists, Inc. All rights reserved.

23

Page 24: 2015 Pharmacotherapy Specialty Examination Review Course ...elearning.ashp.org/Files/Org/c52850f8e2e14ca8b14b... · prepare participants to answer similar questions on the pharmacotherapy

Pain Management 1. World Health Organization. Cancer pain relief: with a guide to opioid availability. 2nd ed. Geneva;

1996. Available at http://whqlibdoc.who.int/publications/9241544821.pdf

2. Portenoy RK. Treatment of cancer pain. Lancet. 2011; 377:2236-47. http://www.thelancet.com/journals/lancet/article/PIIS0140-6736(11)60236-5/fulltext (accessed 2014 May 21).

3. Krantz MJ, Martin J, Stimmel B et al. QTc interval screening in methadone treatment. Ann Intern Med. 2009; 150:387-95. http://www.ncbi.nlm.nih.gov/pubmed/19153406 (accessed 2014 May 21).

4. National Comprehensive Cancer Network. NCCN® clinical practice guidelines in oncology: adult

cancer pain. Version 2.2014. Available at: http://www.nccn.org/professionals/physician_gls/pdf/pain.pdf. Referenced with permission from the NCCN Clinical Practice Guidelines in Oncology (NCCN Guidelines®) for Adult Cancer Pain V.2.2014 © National Comprehensive Cancer Network, Inc 2014. All rights reserved. Accessed [February 16, 2015].

5. Portenoy RK, Ahmend E. Principles of opioid use in cancer pain. J Clin Oncol. 2014; 32: 1662-70. 6. Hui D, Bruera E. A personalized approach to assessing and managing pain in patients with cancer. .

J Clin Oncol. 2014; 32: 1640-46. 7. Hershman DL, Lacchetti C, Dworkin RH, et al. Prevention and management of chemotherapy-

induced peripheral neuropathy in survivors of adult cancers: American Society of Clinical Oncology clinical practice guideline. J Clin Oncol. 2014; 32:1941-67.

Breast Cancer 1. National Comprehensive Cancer Network. NCCN® clinical practice guidelines in oncology: breast

cancer. Version 1.2015. Available at: http://www.nccn.org/professionals/physician_gls/pdf/breast.pdf. Referenced with permission from the NCCN Clinical Practice Guidelines in Oncology (NCCN Guidelines®) for Breast Cancer V.1.2015 © National Comprehensive Cancer Network, Inc 2015. All rights reserved. Accessed [February 22, 2015].

2. Burnstein HJ, Temin S, Anderson H et al. Adjuvant endocrine therapy for women with hormone receptor-positive breast cancer: American Society of Clinical Oncology clinical practice guideline focused update.. J Clin Oncol. 2014; 32:2255-69.

3. Ellis GK, Livingston, RB, Gralow JR et al. Dose-dense anthracycline-based chemotherapy for node-positive breast cancer. J Clin Oncol. 2002; 20:3637-43. http://jco.ascopubs.org/content/20/17/3637.long (accessed 2014 May 21).

4. Romond EH, Perez EA, Bryant J et al. Trastuzumab plus adjuvant chemotherapy for operable HER2-positive breast cancer. N Engl J Med. 2005; 353:1673-84. http://www.nejm.org/doi/full/10.1056/NEJMoa052122 (accessed 2014 May 21).

______________________________________________________________________________________________ ©2015 American Society of Health-System Pharmacists, Inc. All rights reserved.

24

Page 25: 2015 Pharmacotherapy Specialty Examination Review Course ...elearning.ashp.org/Files/Org/c52850f8e2e14ca8b14b... · prepare participants to answer similar questions on the pharmacotherapy

5. Irvin W Jr, Muss HB, Mayer DK. Symptom management in metastatic breast cancer. Oncologist. 2011; 16:1203-14. http://www.ncbi.nlm.nih.gov/pubmed/21880861 (accessed 2014 May 21).

6. Swain SM, Basegla J, Kim SJ, et al. Pertuzumab, transtuzumab, and docetaxel in HER2 positive

metaststic breast cancer. N Engl J Med. 2015; 372: 724-34. 7. Giordano SH, Temin S, Kirshner JJ, et al. Systemic therapy for patients with advanced human

epidermal growth factor receptor 2-positive breast cancer: American Society of Clinical Oncology practice guideline. J Clin Oncol. 2014; 32: 2078-99.

8. Partridge AH, Rumble RB, Carey AL, et al. Chemotherapy and targeted therapy for women with

human epidermal growth factor receptor-negative (or unknown) advanced breast cancer: American Society of Clinical Oncology practice guideline. J Clin Oncol. 2014; 32: 3307-29.

______________________________________________________________________________________________ ©2015 American Society of Health-System Pharmacists, Inc. All rights reserved.

25

Page 26: 2015 Pharmacotherapy Specialty Examination Review Course ...elearning.ashp.org/Files/Org/c52850f8e2e14ca8b14b... · prepare participants to answer similar questions on the pharmacotherapy

Safety in Cancer Patients 1. Verma S, Madarnas Y, Shedev S et al. Patient adherence to aromatase inhibitor treatment in the

adjuvant setting. Current Oncology. 2011; 18(Suppl 1):S3-9. http://www.ncbi.nlm.nih.gov/pmc/articles/PMC3119895/ (accessed 2014 May 21)

2. Holbrook AM, Pereira JA, Labiris R et al. Systemic overview of warfarin and its drug and food

interactions. Arch Intern Med. 2005; 165:1095-106. http://www.ncbi.nlm.nih.gov/pubmed/15911722 (accessed 2014 May 21).

3. Schulmeister L. Extravasation management: clinical update. Semin Oncol Nurs. 2011; 27:82-90. http://www.ncbi.nlm.nih.gov/pubmed/21255716 (accessed 2014 May 21).

4. Senkus E, Jassem J. Cardiovascular effects of systemic cancer treatment. Cancer Treat Rev. 2011; 37:300-11. http://www.ncbi.nlm.nih.gov/pubmed/21126826 (accessed 2014 May 21).

Patient Resources 1. Jefford M, Tattersall MHN. Informing and involving cancer patients in their own care. Lancet Oncol.

2002; 3:629-37. http://www.thelancet.com/journals/lanonc/article/PIIS1470-2045(02)00877-X/abstract (accessed 2014 May 21).

2. NCCN® Guidelines for Patients (http://www.nccn.org/patients/default.asp). Referenced with permission from the NCCN Clinical Practice Guidelines in Oncology (NCCN Guidelines®) for Patients © National Comprehensive Cancer Network, Inc 2014. All rights reserved. Accessed [November 19, 2014].

3. ASCO Website for Patients (http://www.cancer.net)

4. American Cancer Society (http://www.cancer.org/)

5. LIVESTRONG Foundation (http://www.livestrong.org/)

6. Clinical Trials (www.clinicaltrials.gov) 7. eHealth Initiative: Health IT Cancer Resource Guide (http://ehidc.org/resource-

center/directories/hit-cancer-resource-guide)

______________________________________________________________________________________________ ©2015 American Society of Health-System Pharmacists, Inc. All rights reserved.

26

Page 27: 2015 Pharmacotherapy Specialty Examination Review Course ...elearning.ashp.org/Files/Org/c52850f8e2e14ca8b14b... · prepare participants to answer similar questions on the pharmacotherapy

Complex Breast Cancer Case

Helen M. Marshall, Pharm.D., BCPS, BCOPClinical Pharmacist, Hematology/Oncology

Seattle Cancer Care Alliance ‐ University of Washington Medical Center

Clinical Assistant Professor

University of Washington School of Pharmacy

Seattle, Washington

• I have nothing to disclose related to the content of this presentation.

Disclosure

Learning Objectives

• Correctly answer case‐based questions about appropriate treatment and monitoring of a complex patient with multiple conditions, including breast cancer, febrile neutropenia, and pulmonary embolism.

• Develop a plan to manage pain and nausea/vomiting in a patient with cancer.

• Determine how to manage drug‐drug and drug‐disease interactions in a cancer patient.

Learning Objectives

• Discuss safety issues in this population.

• Identify and recommend appropriate resource organizations/groups to assist a specific patient.

Our Patient (HPI)

• GE is a 53 year old female being treated for breast cancer who presents to the ED with acute onset chest pain and shortness of breath

• GE reports that she has been vomiting for the last 24 hours and is unable to eat

• She also feels dizzy and weak

• Vitals: RR 25, BP 90/52, HR 105, Temp 98.4°F 

• Wt 76kg, Ht 166 cm

PMH

• Hypertension

• Depression

• Hypothyroidism

• Breast Cancer, Stage IIB

Diagnosed January 2015 in her right breast

Stage IIB (T2 N1 M0)

ER and PR positive

HER2 negative

Postmenopausal (menopause at age 51)

______________________________________________________________________________________________ ©2015 American Society of Health-System Pharmacists, Inc. All rights reserved.

27

Page 28: 2015 Pharmacotherapy Specialty Examination Review Course ...elearning.ashp.org/Files/Org/c52850f8e2e14ca8b14b... · prepare participants to answer similar questions on the pharmacotherapy

Breast Cancer Treatment to Date

• Surgery

Mastectomy with axillary lymph node dissection

4 positive lymph nodes

• Radiation

Chest wall and regional lymph nodes

Breast Cancer Treatment to Date• Chemotherapy

Dose‐dense AC (Adriamycin[doxorubicin] and cyclophosphamide)

—Doxorubicin 60 mg/m2 IV day 1

—Cyclophosphamide 600 mg/m2 IV day 1

—Every 14 days X 4 cycles

—Dose‐dense AC requires growth factor support

Followed by paclitaxel

—Paclitaxel 80 mg/m2 IV day 1 weekly X 12 cycles

• Prevention of nausea and vomiting

Ondansetron 8 mg IV pre‐chemotherapy day 1

Dexamethasone 10 mg IV pre‐chemotherapy day 1

• Our patient received Cycle 2 of doxorubicin and cyclophosphamide 2 days ago

Question 1: Which of the following should be monitored for our patient on this chemotherapy regimen? 

a. Cognitive function

b. Pulmonary function

c. Cardiac function

d. Thyroid function

Evaluating and Monitoring Chemotherapy 

• Patient

• Regimen

• Organ function

• Numbers

• Toxicity

Adverse effects

Supportive care

• Drug‐drug interactions

Medications

• No Known Drug Allergies

• Hydrochlorothiazide 25 mg po daily

• Lisinopril 20 mg po daily

• Multivitamin po daily

• Calcium citrate 500 mg / Cholecalciferol 400 units po twice daily

• Paroxetine 30 mg po daily

• Levothyroxine 88 mcg po daily

Medications

• Chemo and premedication

• Ondansetron 8 mg po every 8 hours prn nausea and vomiting (N/V)

• Prochlorperazine 10 mg po every 6 hours prn N/V

• Lorazepam 0.5‐1 mg po every 6 hours prn N/V, anxiety

______________________________________________________________________________________________ ©2015 American Society of Health-System Pharmacists, Inc. All rights reserved.

28

Page 29: 2015 Pharmacotherapy Specialty Examination Review Course ...elearning.ashp.org/Files/Org/c52850f8e2e14ca8b14b... · prepare participants to answer similar questions on the pharmacotherapy

Labs• CK Index 1• CK‐MB 1 ng/ml• Total CK 41 ng/ml• Troponin 0.2 ng/ml

• WBC 2.7 thou/microL• Hgb 11.2 g/dL• Hct 30%• Plts 96 thou/microL• ANC 0.9 thou/microL

• Na 129 mEq/L• K 3.3 mEq/L• Cl 100 mEq/L• CO2 32 mEq/L• BUN 25 mg/dL• SCr 1.2 mg/dL• Glucose 95 mg/dL

• AST 27 units/L• ALT 18 units/L• Alk Phos 105 units/L• T bili 0.8 mg/dL• TP 7.1 g/dL• Albumin 3.2 g/dL

• 0.9% NaCl IV initiated at 500 ml/hour X 2 liters

• Ondansetron 8 mg IV given X 1

• Dexamethasone 10 mg IV given X 1

• Prochlorperazine 10 mg po Q 6 hr and lorazepam 1 mg PO/IV Q 4‐6 hr are also ordered by the resident on a PRN basis

• 1 hour later, vitals are as follows:

RR 19, BP 115/78, HR 98, Temp 98.4°F

• Spiral CT reveals bilateral pulmonary embolism

Small pulmonary embolus in the distal right lower lobe arterial tree

Large pulmonary embolus in the left upper lobe

Treatment Plan

Question 2: Which of the following are risk factors for venous thromboembolism (VTE) in GE?

a. Febrile neutropenia

b. Nausea and vomiting

c. Presence of Central Venous Catheter 

d. Hypertension

VTE in Malignancy

• Endothelial injury

Central catheter

Chemotherapy

Surgery

• Circulatory Stasis

Prolonged bed rest

Vascular invasion by tumor cells

• Hypercoagulable state

Procoagulants from tumor

Release of cytokines

Direct cell‐to‐cell interaction between endothelial cells, leukocytes and platelets

Question 3:  Which of the following therapies should be initiated in GE for her pulmonary embolism?

a. Warfarin 5 mg orally daily

b. Apixaban 10 mg orally twice daily

c. Dalteparin 12,500 units subcut every 24 hr

d. Enoxaparin 80 mg subcutevery 12 hr

Low Molecular Weight Heparin (LMWH) vs. a Coumarin (CLOT Study)

• All patients initiated on dalteparin subcut 200 units/kg/day

• N = 336 continued on dalteparin subcut 200 units/kg/day X 1 month then dalteparin subcut 150 units/kg/day X 5 months

• N = 336 received dalteparin subcut X 5‐7 days with a coumarin derivative orally X 6 months with target     INR = 2.5

• Primary outcome: Recurrent DVT and/or PE

• Secondary outcomes: Clinically overt bleeding, death

Lee AYY et al. N Engl J Med. 2003; 349:146‐53.

______________________________________________________________________________________________ ©2015 American Society of Health-System Pharmacists, Inc. All rights reserved.

29

Page 30: 2015 Pharmacotherapy Specialty Examination Review Course ...elearning.ashp.org/Files/Org/c52850f8e2e14ca8b14b... · prepare participants to answer similar questions on the pharmacotherapy

CLOT Results

• Primary outcome

Recurrence rates at 6 months: 8% in dalteparin group vs. 16% in the warfarin group

• Secondary outcomes

No significant difference in the rate of major bleeding (6% in the dalteparin group vs. 4% in the oral anticoagulant group)

Mortality rate at 6 months was 39% vs. 41% in the dalteparin and oral anticoagulant arms

Lee AYY et al. N Engl J Med. 2003; 349:146-53.

Thrombosis in Malignancy:Current Recommendations• Initial Phase (5‐7 days)

Dalteparin 200 units/kg subcut Q 24 hours OR

(Tinzaparin 175 units/kg subcut Q 24 hours OR)

Enoxaparin 1 mg/kg subcut Q 12 hours

• Subacute Phase (3‐6 months)

Dalteparin 200 units/kg subcut Q 24 hours X 1 month then 150 units/kg Q 24 hours OR

(Tinzaparin 175 units/kg subcut Q 24 hours OR)

Enoxaparin 1 mg/kg subcut Q 12 hours or 1.5 mg/kg Q 24 hours

• Chronic Phase (6 months to indefinite)

Continue anticoagulation long term or until malignancy resolves

— LMWH

— Warfarin with INR 2‐3 Kearon C et. al. Chest 2012; 141(2 suppl):419S‐94S. Referenced with permission from the NCCN Clinical Practice Guidelines in Oncology (NCCN Guidelines®) for Venous Thromboembolic Disease 

V.2.2014 © National Comprehensive Cancer Network, Inc 2014. All rights reserved. Accessed [February 16, 2015].

Question 4:  What is the appropriate antiemetic regimen for GE while receiving treatment with dose dense AC?  

a. Ondansetron 8 mg IV and dexamethasone 12 mg IV day 1 followed by dexamethasone 4 mg PO BID days 2‐4

b. Fosaprepitant 150 mg IV, ondansetron 8 mg IV and dexamethasone 12 mg IV day 1 followed by dexamethasone 4 mg PO BID days 2‐4

c. Fosaprepitant 150 mg IV, ondansetron 8 mg IV and dexamethasone 12 mg IV day 1 

d. Palonosetron 0.25 mg IV and dexamethasone 10 mg IV day 1 followed by dexamethasone 4 mg PO BID days 2‐4

Chemotherapy Induced Nausea and Vomiting (CINV)

• Acute Emesis within the first 24 hours after treatment

• Delayed Emesis occurring at least 24 hours after treatment

• Neurotransmitters dopamine, 5‐HT, and substance P play a role in CINV

• Prevention and treatment governed by multiple sets of guidelines MASCC ASCO NCCN®

Hesketh PJ. N Engl J Med. 2008; 358:2482‐94.  

CINV Risk Factors

• Young age

• Female sex

• Expectation of severe nausea

• Chemotherapy

Dose 

Emetogenicity

• History of alcohol consumption (inverse)

Hesketh PJ. N Engl J Med. 2008; 358:2482‐94.

Highly Emetogenic Chemotherapy Agents

MASCC NCCN® ASCO

AGENTS CisplatinMechlorethamineStreptozocinCyclophosphamide > 1500 mg/m2

CarmustineDacarbazine

AC (doxorubicin or epirubicin with cyclophosphamide)Carmustine > 250 mg/m2

CisplatinCyclophosphamide > 1500 mg/m2DacarbazineDoxorubicin ≥ 60mg/m2

Epirubicin . 90 mg/m2

Ifosfamide ≥ 2 g/m2/doseMechlorethamineStreptozocin

CarmustineCisplatinCyclophosphamide ≥ 1500mg/m2

DacarbazineDactinomycinMechlorethamineStreptozocin

Specifics on the AC Regimen

Lists Anthracycline + Cyclophosphamide as its own category and is considered a highly emetogenic regimen

See above Asterisks anthracyclines (moderate level) and  states highly emetogenic when combined with cyclophosphamide

Highly Emetogenic is defined as likely to cause N/V in >90% of patients

Referenced with permission from the NCCN Clinical Practice Guidelines in Oncology (NCCN Guidelines®) for Antiemesis  V.2.2014 © National Comprehensive Cancer Network, Inc 2014. All rights reserved. Accessed [February 16, 2015].

Basch E et al. J Clin Oncol. 2011; 29:4189‐98.Gralla RJ et al. MASCC/ESMO Antiemetic Guidelines. 2013.

______________________________________________________________________________________________ ©2015 American Society of Health-System Pharmacists, Inc. All rights reserved.

30

Page 31: 2015 Pharmacotherapy Specialty Examination Review Course ...elearning.ashp.org/Files/Org/c52850f8e2e14ca8b14b... · prepare participants to answer similar questions on the pharmacotherapy

Prevention of CINV (Highly Emetogenic)

MASCC NCCN® ASCO

Day 1 of Chemo

NK1 Antagonist Fosaprepitant 150 mg IVAprepitant 125 mg PO

Fosaprepitant 150 mg IV*Aprepitant 125 mg PO**

Fosaprepitant 150 mg IVAprepitant 125 mg PO

5HT3 Antagonist See next  slide Palonosetron 0.25mg IV (preferred)

See next  slide

Corticosteroid Dexamethasone 12 mg  IV/PO

Dexamethasone 12 mg IV/PO Dexamethasone 12 mg IV/PO

Subsequent Days

NK1 Antagonist (oral regimen)

Aprepitant 80 mg Days 2 and 3

Aprepitant 80 mg Days 2 and 3

Aprepitant 80 mg Days 2 and 3

Corticosteroid Dexamethasone 8 mg  PO  daily for 3‐4 days

Dexamethasone 8 mg PO Day 2 then 8 mg po BID Days 3‐4*Dexamethasone 8 mg PO Days 2‐4**

Dexamethasone 8 mg IV/PO Days 2‐3 or 2‐4

Additional Recommendations

+/‐ 0.5‐2 mg lorazepam Q 4‐6 hours+/‐ H2 blocker or PPI

Notes Olanzapine based regimen also available

5HT3 Antagonist Options• In all three guidelines

• Dolasetron 100 mg PO ONLY (QT prolongation)

• Palonosetron 0.25 mg IV

• Granisetron 0.01mg/kg IV (1mg), 2 mg PO

• In ASCO and MASCC

• Palonosetron 0.5 mg PO

• Ondansetron 8 mg IV or 0.15mg/kg IV

• In NCCN®

• Ondansetron 16‐24 mg PO or 8‐16 mg IV

• Granisetron 1 mg po BID or transdermal patch 3.1mg/24 hour applied 24‐48 hours pre chemo, max 7 days

• In ASCO

• Ondansetron 8 mg PO BID

• In MASCC

• Ondanseton 16 mg PO (8 mg po BID in randomized studies)

Breakthrough N/V Management• Limited prospective data

• Lack of consensus despite guidelines

• Treatment is largely empiric

• Agents frequently used Benzodiazepines

Phenothiazines

• Consider changing original preventative regimen

• NCCN® states the principle is to add an additional agent from a different drug class to the current regimen

Hesketh PJ. N Engl J Med. 2008; 358:2482‐94.  Referenced with permission from the NCCN Clinical Practice Guidelines in Oncology (NCCN Guidelines®) for Antiemesis  V.2.2014 © National Comprehensive Cancer Network, Inc 2014. All rights reserved. Accessed [February 16, 

2015].

Additional Agents for Breakthrough N/V• Phenothiazines

Prochlorperazine* 10 mg PO every 6 hours

Promethazine* 12.5‐25 mg PO (IV if central line only) every 4 hours

• Others

Haloperidol* 0.5‐2 mg PO/IV every 4‐6 hours

Metoclopramide* 10‐40 mg PO/IV every 4 to 6 hours

Scopolamine transdermal patch every 72 hours

• Atypical Antipsychotics

Olanzapine 10 mg po daily X 3 days

• Benzodiazepine

Lorazepam 0.5 – 2 mg PO/IV every 4‐6 hours

• Cannabinoid

Dronabinol 5‐10 mg PO every 3 or 6 hours

Nabilone 1‐2 mg PO BID

Referenced with permission from the NCCN Clinical Practice Guidelines in Oncology (NCCN Guidelines®) for Antiemesis  V.2.2014 © National Comprehensive Cancer Network, Inc 2014. All rights reserved. Accessed 

[February 16, 2015].

* Monitor for dystonic reactions; treat with diphenhydramine

GE’s ED Course

• GE is initiated on enoxaparin 80 mg subcut Q 12 hours

She demonstrates the ability to give herself injections

GE is counseled on signs and symptoms of over and under anticoagulation

A referral for follow up at the cancer center’s pharmacist‐run anticoagulation clinic is made

—Monitor for heparin‐induced thrombocytopenia (HIT) (platelet checks)

—Assess for symptoms of improvement

GE’s ED Course

• Vitals at hour 3 in the ED

RR 20, BP 130/82, HR 100, Temp 98.1°F

• GE completes the fluid bolus

• She reports resolution of dizziness and chest pain

• No further emesis (one dose of oral prochlorperazine and lorazepam administered)

• Instructed to continue scheduled ondansetron 8 mg po every 8 hours and dexamethasone 4 mg po BID X 2 days

• Follow up in clinic for Cycle 3 of chemotherapy planned with a change in antiemetics

______________________________________________________________________________________________ ©2015 American Society of Health-System Pharmacists, Inc. All rights reserved.

31

Page 32: 2015 Pharmacotherapy Specialty Examination Review Course ...elearning.ashp.org/Files/Org/c52850f8e2e14ca8b14b... · prepare participants to answer similar questions on the pharmacotherapy

GE’s Treatment Continued• GE follows up with the anticoagulation clinic the day after her ED visit

• She receives further education

• Platelet monitoring is initiated every 3 days

• She returns to clinic for Cycle 3 of doxorubicin and cyclophosphamide

Relevant Labs ED visit

(Chemo D3)

Day 7 Day 10 Day 14

(Cycle 3 of AC)

Platelets(thou/microL)

96 89 56 33

WBC(thou/microL)

2.7 3.7

ANC(thou/microL)

0.9 1.5

SCr (mg/dL) 1.1 0.8

Heparin‐Induced Thrombocytopenia (HIT, Type II)• Antibody‐mediated adverse drug reaction

• Thrombotic complications

Pulmonary embolism

Ischemic limb necrosis requiring amputation

Acute myocardial infarction

Stroke

• If risk of HIT is > 1%, monitor platelets every 2‐3 days from Day 4‐14 (or until heparin is stopped)

• Do not start warfarin until platelets have recovered (>150 X 109/L) and use low doses initially (5 mg/day)

Linkins LA et al. Chest. 2012; 141(2 suppl):495S‐530S.

The 4T Score: An example of PreTest Probability Scoring for HIT

Category 2 Points 1 Point 0 Points

ThrombocytopeniaPlatelet count fall > 50% with nadir ≥20 X 109/L

Platelet count fall 30‐50% or nadir 10‐19 X 109/L

Platelet count fall < 30% or nadir < 10 X 109/L

Timing of Fall of PlateletCount

Onset between Days 5 –10 or platelet fall ≤ 1 day (with heparin exposure in last 30 days)

Consistent with days 5–10 fall, but unclear (e.g. missing platelet counts) or onset after day 10 or fall ≤ 1 day (prior heparin exposure 30–100 days ago)

Platelet count fall < 4 days without recent heparin exposure

Thrombosis or Sequelae

New thrombosis (confirmed) or skin necrosis at heparin injection sites or acute systemic reaction after intravenous heparin bolus

Progressive or recurrent thrombosis or nonnecrotizing (erythematous) skin lesions or suspected thrombosis (not proven)

None

Other Causes for Thrombocytopenia

None Possible Definite

The 4T Score

• Total Score

≤ 3 points = Low Probability of HIT

4‐5 points = Intermediate Probability of HIT

≥ 6 points = High Probability of HIT

Lo GK et al. J Thromb Haemost. 2006; 4:759–65.

Question 5: Which of the following are therapeutic options for anticoagulation for suspected HIT?

a. Argatroban

b. Dalteparin

c. Dabigatran

d. Clopidogrel

Direct Thrombin Inhibitors (DTIs)Name Bivalirudin Argatroban

Starting Dose 0.15 mg/kg/hr 2 mcg/kg/min 

Dose Modifications

0.5‐1.2  mcg/kg/min inpts with heartfailure, multi‐organsystem failure,severe anasarca orpost‐cardiacsurgery

Dosing in Renal Impairment Reduction for CrCl < 60 ml/min None

Dosing in Hepatic Impairment NoneInitial dose for moderate impairment is 0.5 mcg/kg/min

Effect on INR Moderate Profound

Kiser TH et al. Pharmacotherapy. 2008 Sep; 28(9):1115‐24. Angiomax (bivalirudin) prescribing information. 

http://www.angiomax.com/downloads/Angiomax_US_PI_June_2013.pdfArgatroban prescribing information.  GlaxoSmithKline; April 2012.

______________________________________________________________________________________________ ©2015 American Society of Health-System Pharmacists, Inc. All rights reserved.

32

Page 33: 2015 Pharmacotherapy Specialty Examination Review Course ...elearning.ashp.org/Files/Org/c52850f8e2e14ca8b14b... · prepare participants to answer similar questions on the pharmacotherapy

Fondaparinux: An Outpatient Option• An outpatient option for patients with malignancy

• Labeling for DVT/PE is in conjunction with warfarin

• Fondaparinux causes inhibition of Factor Xa, and dosing is weight‐based

• Limitations:

Not studied (further study recommended in ACCP guidelines)

Long half‐life

Lacks reversal agent

• Use is restricted to patients with HIT or allergy to heparin/LMWH

Linkins LA et al. Chest. 2012; 141(2 suppl):495S‐530S. 

GE’s HIT Management

• Enoxaparin immediately discontinued

• Initiated on fondaparinux 7.5 mg subcut Q 24 hours

• Continuation of fondaparinux was discussed by the oncologist, but the patient’s copay was over $1000, and she was not eligible for patient assistance (patient paid for a 7‐day supply)

• Initiated on warfarin 5 mg po daily after platelets recovered 

• HIT Antibody was positive

Interval History• 7 days after Cycle 3 of AC chemotherapy, GE calls into clinic with fever 101.2°F

• She reports sweats and chills, cough, and feeling very “worn down”

• She asks to take acetaminophen, but her nurse recommends she come into clinic for evaluation

2 sets of blood cultures drawn (one from port, one peripherally), urine cultures obtained

Labs

—BMP, LFTs WNL

—CBC: WBC 0.9 thou/microL, Hgb 10.8 g/dL, Hct 27%, Plts 79 thou/microL, ANC 0.38 thou/microL

Vitals RR 19, BP 125/84, HR 97, Temp 101.4°F 

Question 6:  Which of the following should be initiated for empiric therapy for febrile neutropenia for GE?

a. Imipenem 500 mg IV every 6 hours

b. Ciprofloxacin 500 mg PO every 8 hours and clindamycin 450 mg PO every 8 hours 

c. Ciprofloxacin 500 mg PO every 8 hours and amoxicillin/clavulanate 500 mg PO every 8 hours

d. Ciprofloxacin 500 mg IV every 8 hours

Febrile Neutropenia (FN)• Definition

ANC <0.5 X 109/L  or an ANC that is expected to decrease to <0.5 X 109/L during the next 48 hours and 

A single oral temperature >101°F (38.3°C) or >100.4° F (>38°C) for at least 1 hour

• Common source of cancer‐related morbidity and mortality

• Fever is only reliable indicator of infection Beware of masking fever with acetaminophen (APAP)/NSAIDs

• Primary sites of infection GI tract Lungs, sinuses Skin

Freifeld AG et al. Clin Infect Dis. 2011; 52:ee56‐93. Referenced with permission from the NCCN Clinical Practice Guidelines in Oncology (NCCN 

Guidelines®) for Prevention and Treatment of Cancer‐Related Infections V.2.2014 © National Comprehensive Cancer Network, Inc 2014. All rights reserved. Accessed [February 16, 2015].

Risk Factors for FN

• Age

• Performance status

• Chemotherapy dose intensity

• Nutritional status

• Low baseline blood counts

Lyman GH et al. Oncologist. 2005; 10:427‐37. 

______________________________________________________________________________________________ ©2015 American Society of Health-System Pharmacists, Inc. All rights reserved.

33

Page 34: 2015 Pharmacotherapy Specialty Examination Review Course ...elearning.ashp.org/Files/Org/c52850f8e2e14ca8b14b... · prepare participants to answer similar questions on the pharmacotherapy

Treatment of FN

• Immediate 

• Utilize bactericidal agents

• Design guidelines based on your antibiogram and antimicrobial cost

• Utilize the MASCC, IDSA, and NCCN® guidelines to guide therapy

Referenced with permission from the NCCN Clinical Practice Guidelines in Oncology (NCCN Guidelines®) for Prevention and Treatment of Cancer‐Related Infections V.2.2014 © National Comprehensive Cancer Network, 

Inc 2014. All rights reserved. Accessed [February 16, 2015].Freifeld AG et al. Clin Infect Dis. 2011; 52:e56‐93.Klastersky J et al. J Clin Oncol. 2000; 18:3038‐51.

Guideline Comparison: FN Risk Stratification MASCC Scoring Index Characteristic Score NCCN® Low Risk

Extent of Illness No Symptoms 5

Mild Symptoms 5

Moderate Symptoms 3

SBP ≥ 90 mm Hg 5

No COPD 4 No acute comorbid illness

Score ≥ 21  indicates low risk

Solid tumor (or heme malignancy w/o previous fungal infection)

4Anticipated short duration of neutropenia (<0.1 X109/L < 7 days)

No dehydration 3 No hepatic or renal insufficiency

Outpatient at fever onset 3 Outpatient at fever onset

Age < 60 yr 2 ECOG Performance Status 0‐1

Referenced with permission from the NCCN Clinical Practice Guidelines in Oncology (NCCN Guidelines®) for Prevention and Treatment of Cancer‐Related Infections V.2.2014 © National Comprehensive Cancer Network, Inc 2014. All rights reserved. Accessed [February 16, 2015].

Klastersky J et al. J Clin Oncol. 2000; 18:3038‐51. Freifeld AG et al. Clin Infect Dis. 2011; 52:e56‐93.

Guideline Comparison: FN Risk Stratification MASCC Scoring Index Characteristic Score NCCN® Low Risk

Extent of Illness No Symptoms 5

Mild Symptoms 5

Moderate Symptoms 3

SBP ≥ 90 mm Hg 5

No COPD 4 No acute comorbid illness

Score ≥ 21  indicates low risk

Solid tumor (or heme malignancy w/o previous fungal infection)

4Anticipated short duration of neutropenia (<0.1 X109/L < 7 days)

No dehydration 3 No hepatic or renal insufficiency

Outpatient at fever onset 3 Outpatient at fever onset

Age < 60 yr 2 ECOG Performance Status 0‐1

Referenced with permission from the NCCN Clinical Practice Guidelines in Oncology (NCCN Guidelines®) for Prevention and Treatment of Cancer‐Related Infections V.2.2014 © National Comprehensive Cancer Network, Inc 2014. All rights reserved. Accessed [February 16, 2015].

Klastersky J et al. J Clin Oncol. 2000; 18:3038‐51. Freifeld AG et al. Clin Infect Dis. 2011; 52:e56‐93.

ASCO Guidelines: Antimicrobial Prophylaxis & Outpatient Management of Fever and Neutropenia

PROPHYLAXIS OUTPATIENT FN• Only use antibacterial and antifungal 

prophylaxis if neutrophils are expected to remain <100/µL for > 7 days (unless other factors increase risk for complications/mortality)

Oral fluoroquinolone

Oral triazole

• Interventions such as neutropenic diet, surgical/respiratory masks, supplements are not recommended (no data)

• Assess risk for medical complications in patients with FN

MASCC score (≥21)

Talcott’s rules (Talcott group 4)

No additional risk factors

• Oral fluoroquinolone plus amoxicillin/clavulanate (clindamycin for PCN allergic)

Unless FQ prophylaxis used prior to fever development

Give within 1 hour of triage

• Monitor patient for 4 hours to determine suitability of outpatient management

In 2014, NCCN® added moxifloxacin as a Category 1 recommendation for low risk FN treatment. Note moxifloxacin lacks Pseudomonas coverage.

Treatment of FN: High Risk

• Monotherapy

Anti‐Pseudomonal penicillin +/‐ ‐lactamase inhibitor or 

Extended‐spectrum cephalosporin or

Carbapenem

• Combination therapy

Aminoglycoside + anti‐Pseudomonal penicillin +/‐ ‐lactamase inhibitor

Aminoglycoside + extended‐spectrum cephalosporin

Ciprofloxacin + anti‐Pseudomonal penicillin

Referenced with permission from the NCCN Clinical Practice Guidelines in Oncology (NCCN Guidelines®) for Prevention and Treatment of Cancer‐Related Infections V.2.2014 © National Comprehensive Cancer Network, Inc 

2014. All rights reserved. Accessed [February 16, 2015].Freifeld AG et al. Clin Infect Dis. 2011; 52:e56‐93.

Question 7: What factors for GE should be considered when deciding if vancomycin should be added to GE’s regimen?

a. All FN patients should receive vancomycin

b. Presence of a port

c. Prior ciprofloxacin prophylaxis

d. Reported symptom of cough

______________________________________________________________________________________________ ©2015 American Society of Health-System Pharmacists, Inc. All rights reserved.

34

Page 35: 2015 Pharmacotherapy Specialty Examination Review Course ...elearning.ashp.org/Files/Org/c52850f8e2e14ca8b14b... · prepare participants to answer similar questions on the pharmacotherapy

FN Treatment: Addition of Vancomycin

• 15 mg/kg IV Q12 hours should be added if Risk factor for Streptococcus viridans

—Prophylaxis (fluoroquinolone or trimethoprim‐sulfamethoxazole)

—Severe mucositis

Colonization with resistant Streptococci or Staphylococci Catheter‐related infection Blood culture positive for gram‐positive bacteria Hypotension/sepsis Skin or Soft‐tissue infection Pneumonia

• Reassess therapy 24‐72 hours after initiation

Referenced with permission from the NCCN Clinical Practice Guidelines in Oncology (NCCN Guidelines®) for Prevention and Treatment of Cancer‐Related Infections V.2.2014 © National Comprehensive Cancer Network, Inc 2014. All rights reserved. Accessed [February 16, 2015].

Freifeld AG et al. Clin Infect Dis. 2011; 52:e56‐93.

FN Treatment: Addition of Antifungal Coverage

• Most invasive fungal infections are due to Candida or Aspergillosis

• Fungal infections are uncommon early in FN such that treatment is usually delayed

• Candida Therapy

NCCN® – echinocandins

IDSA – insufficient evidence for one agent (FN guidelines); amphotericin B is the “old standard”

• Aspergillosis Therapy

NCCN® ‐ voriconazole

IDSA – voriconazole

• IDSA Guidelines available for Aspergillosis (2008) and Candidiasis(2009)

Referenced with permission from the NCCN Clinical Practice Guidelines in Oncology (NCCN Guidelines®) for Prevention and Treatment of Cancer‐Related Infections V.2.2014 © National Comprehensive Cancer Network, Inc 2014. All rights reserved. Accessed [February16, 2015].

Freifeld AG et al. Clin Infect Dis. 2011; 52:e56‐93.Pappas PG et al. Clin Infect Dis. 2009; 48:503‐35.   Walsh TJ. Clin Infect Dis. 2008; 46:327‐60.

Approach to a Port‐Related Bloodstream Infection

Mermel LA et al. Clinical practice guidelines for the diagnosis and management of intravascular catheter‐related infection: 2009 update by the Infectious Disease Society of America. Clin Infect Dis. 2009; 49:1‐45. 

Clinical infectious diseases: an official publication of the Infectious Diseases Society of America by INFECTIOUS DISEASES SOCIETY OF AMERICA . Reproduced with permission of Oxford.

Follow Up for GE

• GE completed her course of oral antibiotics without complications, no organisms were grown on culture

• Successfully completes 12 cycles of weekly paclitaxelwithout incidence

• Initiated on adjuvant anastrozole 1 mg PO daily (planfor 5 years of therapy)

• Warfarin was discontinued after 6 months ofanticoagulation

Interval History

• 2 months into therapy with anastrozole, GE returns toher oncologist complaining of severe muscle and jointpain

• The oncologist initiates her on naproxen 500 mg PO BID

• GE returns to clinic in 1 week without relief.  She has notbeen working due to pain and spends 75% of her day in bed

• GE is switched to letrozole 2.5 mg po daily, without relieffrom severe arthralgias and myalgias

• She is prescribed oxycodone 5 mg orally every 4‐6 hours prn pain

Question 8: GE’s oncologist asks you for a conversion to a long‐acting opioid as GE has escalated her oxycodone use to 5 mg every 3 hours around the clock and is still reporting poor pain control with inability to return to work.  Which of the following is the best regimen for GE?

a. Fentanyl TDS 100 mcg/hrevery 72 hours

b. Morphine SR 45 mg po Q 8 hours

c. Morphine SR 15 mg po Q 12 hours

d. Morphine SR 30 mg po Q 12 hours

Enlarged slide at back of handout.

______________________________________________________________________________________________©2015 American Society of Health-System Pharmacists, Inc. All rights reserved.

35

Page 36: 2015 Pharmacotherapy Specialty Examination Review Course ...elearning.ashp.org/Files/Org/c52850f8e2e14ca8b14b... · prepare participants to answer similar questions on the pharmacotherapy

Pain

• “an unpleasant sensory and emotional experience associated with actual or potential tissue damage, or described in such terms”

International Association for the Study of Pain 1994

• Whatever the patient says it is

Prevalence of Types of Cancer Pain (Pathophysiology)

Somatic 32‐35%

Visceral 15‐17%

Neuropathic 8‐9%

Somatic & Visceral 11‐13%

Somatic & Neuropathic 21‐23%

Visceral & Neuropathic 2‐4%

Breakthrough 65%

Chang VT et al. J Palliat Med. 2006; 9:1414‐34.

World Health Organization (WHO) Principles of Pharmacologic Management

• By the mouth

• By the clock

• By the ladder

• For the individual

• With attention to detail

World Health Organization 1996; 2nd Edition.

Adapted from WHO Pain Relief Ladder.Palliative Pharmacy Care edited by Jennifer Strickland, published in 2009 by ASHP.

“Strong” opioids+/‐ APAP/NSAIDs+/‐ adjuvants

“Weak” opioids+/‐ APAP/NSAIDs+/‐ adjuvants

Acetaminophen+/‐ NSAIDs+/‐ adjuvants

APAP = acetaminophen

“Weak” opioids: e.g., codeine, hydrocodone, oxycodone/APAP“Strong” opioids: e.g., morphine, hydromorphone, oxycodone

WHO Pain Relief Ladder

Therapy Options• Non‐Opioid

Acetaminophen (APAP)

NSAIDs

COX‐2 Selective Agents

• Opioids

See upcoming slides

• Adjuvants

Note: Use is off‐label Antidepressants Anticonvulsants Consider risk of suicide

Local Anesthetics Bisphosphonates Calcitonin Corticosteroids Hydroxyzine

Pereira A et al. Pain 2013; 154:345‐9.

Opioids: What to UseMild‐Moderate Pain

• Hydrocodone/APAP*

• Oxycodone/APAP*

• Codeine

• *Combo products have variable amounts of opioid

Severe Pain

• Morphine

• Oxycodone

• Hydromorphone

• Methadone

• Fentanyl

• Oxymorphone

______________________________________________________________________________________________ ©2015 American Society of Health-System Pharmacists, Inc. All rights reserved.

36

Page 37: 2015 Pharmacotherapy Specialty Examination Review Course ...elearning.ashp.org/Files/Org/c52850f8e2e14ca8b14b... · prepare participants to answer similar questions on the pharmacotherapy

Opioids: Essential Considerations

• Medication allergies

• Previous opioid exposure and preference

• Severity and nature of disease

• Age of patient

• Extent of cancer, particularly hepatic and renalinvolvement altering opioid pharmacokinetics

• Comorbid disease states

• Dosage form and route of administration

Opioid Dose Titration• Patients currently on opioids

No consensus or guidelines available

• Severe pain (7‐10)** Consider increasing dose by 50‐100%

• Moderate pain (4‐6)** Consider increasing dose by 25‐50%

• Minimal pain (1‐3)** No increase in dose if controlled with current breakthrough pain (BTP) medication

Consider increasing dose by 25%

• End‐of‐dose failure Shorten dosing interval

**On a 10‐point scale with 10 for the worst pain imaginable for the three severity ratings on this slide

Opioids: Equianalgesic Dosing

• Incomplete cross tolerance between opioids

• Wide interindividual variability

• Several different equianalgesia charts available Derived from single‐dose studies

Do not reflect long‐term opioid exposure or changes made due to ineffective analgesia or toxicity

• If pain poorly controlled, use aggressive conversion 75‐100% of dose

• If pain well controlled, use conservative conversion 50‐75% of dose

Opioids: Equianalgesic DosingOPIOID ORAL DOSE PARENTERAL DOSE

Codeine 200 mg N/A

Hydrocodone 30 ‐ 45 mg N/A

Oxycodone 15‐20 mg N/A

Morphine 30 mg 10 mg

Hydromorphone 7.5 mg 1.5 mg

Methadone* * *

Fentanyl** N/A 100 mcg

Oxymorphone 10 mg 1 mg

*Ratio may range from 1:1 at low doses of oral morphine up to 20:1 for patients receiving high doses of oral morphine (~300mg/day)**Fentanyl transdermal systems 25mcg/hr  60mg PO morphine 

Referenced with permission from the NCCN Clinical Practice Guidelines in Oncology (NCCN Guidelines®) for Adult Cancer Pain V.2.2014 © National Comprehensive Cancer Network, Inc 2014. All rights reserved. Accessed 

[November 19, 2014].

Stepwise Approach to Conversion

1. Comprehensive pain assessment

Current level of pain control

Functional status

2. Determine total daily opioid use

3. Use conversion table to convert to new opioid               

4. Individualize dose

More aggressive conversion for uncontrolled pain

Less aggressive conversion for well controlled pain

5. Follow‐up and continual reassessment 

a. Fentanyl TDS 100 mcg/hr every 72 hours

b. Morphine SR 45 mg po Q 8 hours

c. Morphine SR 15 mg po Q 12 hours

d. Morphine SR 30 mg po Q 12 hours

Question 8 Revisited:GE’s oncologist asks you for a conversion to a long acting opioid as GE has escalated her oxycodone use to 5 mg every 3 hours around the clock and is still reporting poor pain control with an inability to return to work. Which of the following is the best regimen for GE?

______________________________________________________________________________________________ ©2015 American Society of Health-System Pharmacists, Inc. All rights reserved.

37

Page 38: 2015 Pharmacotherapy Specialty Examination Review Course ...elearning.ashp.org/Files/Org/c52850f8e2e14ca8b14b... · prepare participants to answer similar questions on the pharmacotherapy

Question 9: GE’s oncologist follows your advice and initiates the long acting pain regimen that was recommended. Which of the following is the best BTP regimen for her?

a. Morphine IR 15 mg po every 1‐2 hours

b. Morphine IR 15 mg po every 4‐6 hours

c. Morphine IR 30 mg po every 6 hours

d. Oxycodone 2.5 mg po every 6 hours

Breakthrough Pain (BTP)

• A fluctuation in pain intensity that interrupts a tolerable background pain.

• 3 types Idiopathic

Incident (predictable and unpredictable)

End‐of‐dose failure

• Characterized by Moderate to severe intensity

Rapid onset (<3 minutes)

Relatively short duration (< 1 hour)

Frequency averaging 1‐6 episodes per day

Breakthrough Pain• The Ideal BTP Medication

Rapid onset (immediate release)

Easy to titrate

Short duration

Cost effective

Minimal side effects

• PO Dose: 10‐20% of daily dose q3hr PRN, OR

• PRN dose given as immediate release product during a given interval equals the total scheduled dose of a sustained release product given in the same interval e.g., morphine SR 90mg PO Q12h = morphine IR 30mg PO q4h 

Cancer‐Related Neuropathic Pain

• NCCN® Guidelines

Antidepressants and anticonvulsants are 1st line adjuvant medications

Derived from data in non‐cancer patients

• ASCO Guidelines for CIPN

No agents recommended for prevention

Moderate recommendation for treatment with duloxetine

• N=231

• 5 week treatment course: duloxetine 30 mg X 1 week then 60 mg X 4 weeks vs. placebo

• Primary outcome: average pain intensity (0‐10 scale)

Duloxetine arm: mean decrease in average pain of 1.06 (95% CI, 0.72‐1.40) vs. placebo arm: 0.34 (95% CI, 0.01‐0.66) (P = .003; effect size, 0.513)

CIPN: Chemotherapy-induced peripheral neuropathyHershman DL, et al. J Clin Oncol 2014; 32:1941-1967.

Smith EM et al. JAMA. 2013 Apr 3;309(13):1359-67.National Comprehensive Cancer Network. NCCN® clinical practice guidelines in oncology: adult cancer pain. Version 2.2014.

Referenced with permission from the NCCN Clinical Practice Guidelines in Oncology (NCCN Guidelines®) for Adult Cancer Pain V.2.2014 © National Comprehensive Cancer Network, Inc 2014. All rights reserved. Accessed [February 16, 2015].

GE update

• Completes 5 years of therapy with letrozole

• GE is then initiated on tamoxifen 20 mg PO once daily (plan for 5 years of therapy)

• GE tapers off pain medications over a period of 6 months

Question 10: Which of GE’s medications poses a concern if initiating therapy with tamoxifen?

a. Lisinopril

b. Levothyroxine

c. Paroxetine

d. Morphine

______________________________________________________________________________________________ ©2015 American Society of Health-System Pharmacists, Inc. All rights reserved.

38

Page 39: 2015 Pharmacotherapy Specialty Examination Review Course ...elearning.ashp.org/Files/Org/c52850f8e2e14ca8b14b... · prepare participants to answer similar questions on the pharmacotherapy

Breast Cancer Epidemiology

• 231,840 new cases of invasive breast cancer expected in 2015

• 40,290 women expected to die from breast cancer this year

• Incidence rates decreased 7% from 2002‐2003

Women’s Health Initiative published in 2002

Decline in use of hormone replacement therapy

Incidence rates stable recently

• 2nd leading cause of cancer death in women

• 2.8 million survivors in the United States

American Cancer Society. Available at: http://www.cancer.org/cancer/breastcancer/detailedguide/breast-cancer-key-statistics. February 16,2015.

Individualized Drug Therapy in Breast Cancer• Menopausal Status

Potential for use of tamoxifen vs. aromatase inhibitors

Need for ovarian suppression

Fertility preservation

• Hormone Receptor Status (ER/PR)

Use of hormonal therapies

• HER2 Status

Use of trastuzumab, lapatinib, pertuzumab or ado‐trastuzumab emtansine

• Patient factors such as performance status, organ function, etc.

Hormonal Therapies

• Nonsteroidal Aromatase Inhibitors

Anastrozole

Letrozole

• Steroidal aromatase inactivator  Exemestane

• Selective Estrogen Receptor Modulators

Tamoxifen

Toremifene

• Estrogen Receptor Antagonist  Fulvestrant

• Others include megestrol acetate, fluoxymesterone, ethinyl estradiol

Palbociclib (Ibrance®)• Indication

In combination with letrozole

Postmenopausal patients with ER+Her2‐metastatic breast cancer (1st line therapy)

• MOA

Inhibits cyclin‐dependent kinase (CDK) 4 & 6

Deceases cell proliferation by blocking cell progression from G1 to S phase

• Dose: 125 mg po daily Days 1‐21 Q 28 days

• Median PFS 20.2 months (95% CI 13.8‐27.5) vs 10.2 months (95% CI5.7‐12.6) with letrozole alone, HR 0.488 (95% CI 0.319‐0.784) Finn RS et al. Lancet Oncol. 2015; 16: 25-35.

Active Antineoplastic Agents in Breast Cancer

Class Agents

Anthracyclines

Doxorubicin

Epirubicin

Liposomal doxorubicin

Taxanes

Paclitaxel

Docetaxel

Albumin‐bound paclitaxel

AntimetabolitesGemcitabine

Capecitabine

Other Microtubule InhibitorsVinorelbine

Eribulin

Other Active AgentsCyclophosphamide, mitoxantrone, cisplatin, carboplatin, etoposide, vinblastine, fluorouracil, ixabepilone

HER2 Targeted Therapy

• Trastuzumab Adjuvant Setting

—as part of a treatment regimen consisting of doxorubicin, cyclophosphamide, and a taxane

—with docetaxel and carboplatin

—single agent following multi‐modality anthracycline‐based therapy

Metastatic Setting—In combination with paclitaxel for 1st line treatment 

—As a single agent for patients who have received one or more chemotherapy regimens for metastatic disease

______________________________________________________________________________________________ ©2015 American Society of Health-System Pharmacists, Inc. All rights reserved.

39

Page 40: 2015 Pharmacotherapy Specialty Examination Review Course ...elearning.ashp.org/Files/Org/c52850f8e2e14ca8b14b... · prepare participants to answer similar questions on the pharmacotherapy

HER2 Targeted Therapy• Lapatinib

With capecitabine, for the treatment of patients who have received prior therapy including an anthracycline, a taxane, and trastuzumab

With letrozole

• Pertuzumab

In combination with trastuzumab and docetaxel for patients with metastatic breast cancer who are treatment‐naïve

Neoadjuvant therapy (with trastuzumab and docetaxel) for early breast cancer

• Ado‐trastuzumab emtansine

As a single agent, for the treatment of patients who previously received trastuzumab and a taxane (separately or in combination)

CLEOPATRA

• Trastuzumab and docetaxel +/‐ pertuzumab in HER2+ metastatic breast cancer with no prior treatment

• Median overall survival was 56.5 months (95% CI, 49.3 to not reached) in the pertuzumab arm, compared with 40.8 months (95% CI, 35.8 to 48.3) in the placebo arm (HR favoring the pertuzumab group, 0.68; 95% CI, 0.56 to 0.84; P<0.001)

• 15.7 months difference

Swain SM, et al. N Engl J Med. 2015; 372:724-34.

Question 11: Which of these therapies is a possible treatment options for GE if her cancer returns (metastatic to the bone) during treatment with tamoxifen? 

a. Exemestane with everolimus

b. Ado‐trastuzumabemtansine

c. Pertuzumab, trastuzumaband paclitaxel

d. Tamoxifen (complete 5 years)

Follow‐up Therapy for Endocrine Treatment of Recurrent or Stage IV Disease

• Continue endocrine therapy until progression or unacceptable toxicity

• If no clinical benefit after 3 sequential endocrine regimens OR

• Symptomatic visceral disease 

Proceed with chemotherapy

Referenced with permission from the NCCN Clinical Practice Guidelines in Oncology (NCCN Guidelines®) for Breast Cancer V.1.2015 © National Comprehensive Cancer Network, Inc 2015. All rights reserved. Accessed [February 22, 2015].

Examples of Safety Issues in Our Patient Case

• Medication Adherence

Hormonal therapies

• Warfarin drug‐drug interactions

Consider influence of active cancer and chemotherapy on bleeding risk

• Extravasation

GE’s need for a central venous catheter

• Cardiotoxicity

Anthracycline

Endocrine Therapy Medication Adherence

• Data evaluating adherence are variable

Methods

Definitions

Endpoints

Duration of follow up

• Some studies show up to 50% discontinuation rates of endocrine therapies prior to completion of recommended course

• Discontinuation is linked to increased mortalityHershman DL et al. Breast Cancer Res Treat. 2011; 126: 529‐37.

McCowan C et al. Br J Cancer. 2008; 99:1763‐8.Dezentje VO et al. J Clin Oncol. 2010; 28(14):2423‐9.Hershman DL et al. J Clin Oncol. 2010; 28(27):4120‐8.

______________________________________________________________________________________________ ©2015 American Society of Health-System Pharmacists, Inc. All rights reserved.

40

Page 41: 2015 Pharmacotherapy Specialty Examination Review Course ...elearning.ashp.org/Files/Org/c52850f8e2e14ca8b14b... · prepare participants to answer similar questions on the pharmacotherapy

Barriers to Adherence: Adjuvant Aromatase Inhibitor Therapy

Patient‐specific Factors Provider‐related Factors Treatment‐related Factors

Health beliefs (lack of belief in therapy)

Importance of adherence may not be emphasized

Adverse effects

Depression/Antidepressant use Difficulties of long term adherence may not be discussed

Underestimation of side effects by physician

Mastectomy Lack of support for dealing with side effects

Unexpected effects associated with non‐adherence

Poor awareness of therapeutic benefit

Dissatisfaction with provider

Remembering to take medication(Older and younger age groups associated with non‐adherence)

Inconvenience

Difficulty swallowing pills

Verma S et al. Curr Oncol. 2011 May; 18 Suppl 1:S3‐9.

Warfarin Drug Interactions• Too many for this slide!

Substrate of CYP1A2, 2C19, 2C9 (major), and 3A4

Weak inhibitor of CYP2C19 and 2C9

Common agents for cancer patients (e.g., antibiotics, chemotherapy)

• Increased bleeding risk

Thrombocytopenia

Disease related

Chemotherapy

Bleeding tumors

• Intensive monitoring required for cancer patients

Changes in appetite

Fatigue

Nausea/vomiting

Example: increased fall risk due to chemotherapy‐induced peripheral neuropathy

Extravasation• Symptoms can be delayed or immediate

Pain, burning

Swelling

Erythema

Loss of blood return

• Determine if agent binds to DNA for direct vs. indirect effect

• Results in partial or full thickness skin loss

• Minimal evidence base for treatment

Schulmeister L. J Clin Oncol Nurs. 2007; 11(3):387‐95.  Wickham R et al. Oncology Nursing Forum. 2006; 33:1143‐50.

Schrijvers DL. Annals of Oncology. 2003; 14 (supplement 3): iii26‐30.Photos: TOTECT® Case Study, Copyright © 2006 Topo Target USA, Inc. Used with 

permission.    

Treatment of ExtravasationAgent Antidote Comments

Anthracyclines Dexrazoxane1000 mg/m2 IV within 5 hours on Day 1, repeat Day 2 then 500 mg/m2 IV on Day 3

DMSO Apply locally ASAP and repeat Q 8 hours X 7 days

Cold packs Vasoconstriction decreases local drug uptake

Nitrogen Mustards Sodium thiosulfate2 mL of a solution of 4 mL sodium thiosulfate and 6 mL sterile water

Vinca Alkaloids Hyaluronidase 50‐1500 units subcut

Warm Packs Vasodilation to enhance drug uptake

Taxanes Normal saline Exerts dilutional effect

DMSO

Cold packs ?

Surgical intervention may also be warranted

Wickham R et al. Oncology Nursing Forum. 2006; 33:1143‐50.Schrijvers DL.  Annals of Oncology. 2003; 14 (supplement 3): iii26‐30.

Cardiotoxicity

• Anthracycline Induced ‐ Cardiomyopathy

Delayed

Cumulative

Dose‐related (Lifetime maximum dose)

Risk factors include heart disease, HTN, mediastinal radiation, prior anthracycline

• Trastuzumab Induced

Additive with anthracyclines

• Recommendations for monitoring for our patient’s initial chemotherapy regimen

Cardiac monitoring at baseline, 3, 6 and 9 months

Pharmacist’s Role in Complex Oncology Patients

• Drug therapy recommendations

Chemotherapy

Supportive care

• Anticoagulation management

• Pain Management

• Medication safety, especially chemotherapy

• Medication access

• Monitoring 

• Patient and family education

______________________________________________________________________________________________ ©2015 American Society of Health-System Pharmacists, Inc. All rights reserved.

41

Page 42: 2015 Pharmacotherapy Specialty Examination Review Course ...elearning.ashp.org/Files/Org/c52850f8e2e14ca8b14b... · prepare participants to answer similar questions on the pharmacotherapy

Cancer Patient Resources• NCCN® Guidelines for Patients (http://www.nccn.org/patients/default.asp) Referenced with permission from the NCCN Clinical Practice Guidelines in Oncology (NCCN Guidelines®) for Patients © National Comprehensive Cancer Network, Inc 2014. All rights reserved. Accessed [November 19, 2014].

• ASCO Website for Patients (http://www.cancer.net)

• American Cancer Society (http://www.cancer.org/)

• LIVESTRONG (http://www.livestrong.org/)

• Clinical Trials (www.clinicaltrials.gov)

• eHealth Initiative: Health IT Cancer Resource Guide(http://www.ehidc.org/resource‐center/directories/hit‐cancer‐resource‐guide)

______________________________________________________________________________________________ ©2015 American Society of Health-System Pharmacists, Inc. All rights reserved.

42

Page 43: 2015 Pharmacotherapy Specialty Examination Review Course ...elearning.ashp.org/Files/Org/c52850f8e2e14ca8b14b... · prepare participants to answer similar questions on the pharmacotherapy

Approach to a Port‐Related Bloodstream Infection

Mermel LA et al. Clinical practice guidelines for the diagnosis and management of intravascular catheter‐related infection: 2009 update by the Infectious Disease Society of America. Clin Infect Dis. 2009; 49:1‐45. 

Clinical infectious diseases: an official publication of the Infectious Diseases Society of America by INFECTIOUS DISEASES SOCIETY OF AMERICA . Reproduced with permission of Oxford.

______________________________________________________________________________________________ ©2015 American Society of Health-System Pharmacists, Inc. All rights reserved.

43